Appleton and Lange Review Questions

Lakukan tugas rumah & ujian kamu dengan baik sekarang menggunakan Quizwiz!

C. Placenta accreta The "swiss cheese" appearance is referring to large anechoic spaces in and behind the placenta and increased placental vascularity.

"Swiss cheese" appearance retroplacentally is associated with what abnormality? A. Placenta previa B. Placental abruption C. Placenta accreta D. Placental chorioangioma

D. Image fetal face Three-dimensional imaging of the internal organs is termed volume imaging. Three-dimensional images of surface structures, such as the fetal face, is surface rendering.

3-D surface rendering is used to: A. Obtain volume measurements B. Obtain technically difficult images C. Image fetal spine D. Image fetal face

A. Fetal face Three-dimensional imaging of surface structures, such as the fetal face, is termed surface rendering.

3D volumetric reconstructions are used to show all of the following except A. Fetal face B. Fetal limbs C. Organs D. Digits

C. Dysgerminoma Dysgerminomas is one of the most common malignant tumors of pregnancy. Granulosa cell and thecoma tumors typically occur postmenopausally, and fibromas usually occur in the 5-6th decade.

A 24 year old patient presents for an ultrasound. The sonogram shows a solid hypoechoic ovarian mass. The differential is most likely A. Granulosa cell B. Thecoma C. Dysgerminoma D. Fibroma

B. Autosomal dominant polycystic kidney disease Autosomal dominant polycystic kidney disease does not typically cause renal disease prenatally therefore the amniotic fluid is normal. The. Kidneys may appear large and echogenic. Autosomal recessive polycystic kidney disease does affect renal function and is associated with encephaloceles and postaxial polydactyly.

A 26-week fetus presents with large echogenic kidneys that contain some small cysts. The amniotic fluid volume is normal. Differential diagnosis includes A. Autosomal recessive polycystic kidney disease B. Autosomal dominant polycystic kidney disease C. Meckel's syndrome D. A and C E. All of the above

A. Ureterocele A ureterovesicle junction (UVJ) obstruction is an obstruction at the junction of the ureter and fetal bladder. It is associated with ureter anomalies, such as duplication and abnormal insertion sites. There is often an ureterocele caused by the abnormal insertion of the ureter. Hydroureter and mild hydronephrosis are commonly present.

A UVJ obstruction is associated with what other findings? A. Ureterocele B. Unilateral hydronephrosis C. Pelvic kidney D. A and C

A. Tubo-ovarian abscess The majority of tubo-ovarian abscesses are bacterial in origin. Symptoms are similar to other bacterial infections including pain, fever, and increased white blood cell count.

A complex adnexal mass is identified in a patient with tenderness and elevated temperature. The most likely diagnosis is A. Tubo-ovarian abscess B. Corpus luteal cyst C. Serous cystadenoma D. Brenner's tumor

C. Limb-body wall complex Limb-body wall complex (LBWC) is the most severe abdominal wall defect with the entire ventral wall disrupted. If the umbilical cord is visualized at all, it is very short and severe scholiosis is present.

A condition in which no umbilical cord may be identified, along with ventral wall defects, and scoliosis is most likely A. Cloacal exstrophy B. Amniotic synechia C. Limb-body wall complex D. Complete ventral wall defect

C. Wilm's tumor Nephroblastoma, also known as Wilm's tumor, is a malignant renal tumor that sonographically appears similar to a mesoblastic nephroma.

A congenital mesoblastic nephroma is also known as a A. Neuroblastoma B. Teratoma C. Wilm's tumor D. William's tumor

A. Allantoic cyst A true cord cyst is attributable to allantoic duct remnants and is thought to be more common in the first trimester.

A cyst in the umbilical cord is called? A. Allantoic cyst B. Yolk sac cyst C. Meckel's cyst D. Ectodermal cyst

B. False The only definitive test for any chromosomal abnormality is karyotyping of fetal cells.

A definitive diagnosis of trisomy 18 can be made by ultrasound if the fetus has multiple structural abnormalities. A. True B. False

C. A boy XY karyotype indicates a male fetus. XX karyotype indicates a female fetus. Down syndrome is labeled 47, +21, and Turner syndrome in labeled 45X

A fetal karyotype is 46XY. This fetus is A. Affected with Down syndrome B. Affected with Turner syndrome C. A boy D. A girl

B. Affected with Turner syndrome XY karyotype indicates a male fetus. XX karyotype indicates a female fetus. Down syndrome is labeled 47, +21, and Turner syndrome is labeled 45X

A fetal karyotype is 47X. This fetus is A. Affected with Down syndrome B. Affected with Turner syndrome C. A boy D. A girl

D. AV malformation A Vein of Galen aneurysm is a type of AV malformation. Agenesis of the corpus callosum, third ventricular dilation, and an arachnoid cyst are cystic midline lesions, but they will not have the high, turbulent blood flow of an AV malformation.

A fetus presents with an anechoic midline lesion in the brain, fetal hydrops, and congestive heart failure. The most likely diagnosis for the lesion is A. Agenesis of the corpus callosum B. Dilation of the third ventricle C. Arachnoid cyst D. AV malformation

B. 5 weeks

A gestational sac is identified within the uterus. The CRL measures 3 mm. What is the approximate age of the embryo? A. <4 weeks B. 5 weeks C. 6 weeks D. 8 weeks

B. Ureters will appear predominately anechoic Ureters will typically appear anechioc and tortuous on sonogram. Bowel is typically hypoechoic with peristalsis.

A hydroureter will appear differently than dilated bowel on sonogram because A. Ureters are predominately lateral structures in the fetal abdomen. B. Ureters will appear predominately anechoic C. Bowel will appear predominately anechoic D. Dilated large bowel mainly found in the midline of the fetus.

D. All of the above Identification of a macrosomic fetus can alert the obstetrician to watch for complications of macrosomia during delivery.

A macrosomic fetus is at risk for A. Shoulder dystocia B. Increased perinatal mortality C. Prolonged labor D. All of the above

A. Neuroblastoma A Neuroblastoma appears as a suprarenal mass and should be considered when a mass is identified superior to the kidney.

A malignant adrenal gland tumor that appears as an echogenic, heterogenous mass is A. Neuroblastoma B. Teratoma C. Wilm's tumor D. None of the above

C. Cloacal exstrophy is characterized by an abdominal wall defect inferior to the umbilical cord insertion with exstrophy of a cloacal sac and a neural tube defect. It is associated with a markedly increased MSAFP. Amniotic sheets and congenital diaphragmatic hernia do not increase MSAFP. Smith-Lemli-Opitz syndrome is associated with a low level of maternal uE3 and normal MSAFP.

A markedly increased MSAFP would be associated with A. Amniotic sheets B. Cloacal exstrophy C. Congenital diaphragmatic hernia D. Smith-Lemli-Opitz syndrome

C. Holoprosencephaly Medial cleft lip is associated with a spectrum of midline defects, the most common being holoprosencephaly.

A medial cleft lip has a strong association with what abnormality? A. Hydrocephaly B. Turner syndrome C. Holoprosencephaly D. Hydranencephaly

B. Omphalocele Small omphaloceles may be mistaken for a cord hematoma. If the defect at the base of the cord insertion is >7 mm it is most likely an omphalocele.

A midline defect in the anterior abdominal wall with herniation of abdominal contents into the base of the umbilical cord is called A. Gastroschisis B. Omphalocele C. Cloacal exstrophy D. Pentalogy of Cantrell

G6P3215 The number of pregnancies is listed after gravida (G). The first number after parity (P) is the number of full term pregnancies (3), the second number is the number of preterm pregnancies (1 set of twins, single pregnancy and 1 preterm fetal death >20 weeks = 2), the third number is the number of abortions (1), and the final number is the number of living children (5).

A patient has had six pregnancies: three full-term, one preterm delivery of twins, one spontaneous abortion, one fetal loss at 22 weeks. This would be listed as A. G6P3215 B. G6P3124 C. G7P5411 D. G7P2135

C Hydranencephaly Hydranencephaly will present with no visible cerebral cortex. The differential includes alobar holoprosencephaly (fused thalamus) and severe hydrocephaly (bi-lobed thalamus with dilated third ventricle).

A patient presents for an anatomy scan. The fetal head shows a cranium filled with anechoic fluid, and no cerebral cortex is identified. The brain stem is identified. The most likely diagnosis is A. Semilobar holoprosencephaly B. Alobar holoprosencephaly C. Hydranencephaly D. Hydrocephaly

A. Alobar holoprosencephaly The most likely diagnosis would be alobar holoprosencephaly. Differential diagnosis would include hydranencephaly and hydrocephaly. Hydranencephaly would not show any cerebral cortex and only brain stem would be spared. Hydrocephaly would show a bi-lobed thalamus with a dilated third ventricle.

A patient presents for an anatomy scan. The fetal head shows a single large cystic cavity with a rim of cerebral cortex. A fused thalamus is also identified. The most likely diagnosis is A. Alobar holoprosencephaly B. Semilobar holoprosencephaly C. Hydranencephaly D. Hydrocephaly

A. Semilobar holoprosencephaly Semilobar holoprosencephaly is a single anterior ventricle with partial separation of the posterior cerebellar hemispheres.

A patient presents for an anatomy scan. The fetal head shows a single ventricle, single choroid, two cerebellar hemispheres, and fused thalamus. The abnormality is most likely A. Semilobar holoprosencephaly B. Alobar holoprosencephaly C. Hydranencephaly D. Hydrocephaly

B. Examining the uterus for retained placenta Sonographic assessment of the maternal kidneys and bowel would not be considered a gynecological sonogram.

A postpartum gynecologic sonogram may be needed for A. Assessing maternal hydronephrosis B. Examining the uterus for retained placenta C. Checking for maternal bowel obstruction D. All of the above.

B. False A complete situs inversus will appear with the heart and normally positioned left-sided abdominal organs on the right side. A complete situs inversus has a minimally increased risk of abnormalities and is not as significant as a partial situs inversus.

A quick way to check for organ orientation is to assure that the heart and stomach are on the same side of the fetus. A. True B. False

A. Leiomyosarcoma On ultrasound, a leiomyoma and a leiomyosarcoma appear the same. Clinically, the only difference is a rapid increase in growth in postmenopausal women.

A rapidly growing fibroid in a postmenopausal woman not on hormone replacement therapy could indicate A. Leiomyosarcoma B. Fibrous degeneration C. Hemorrhagic infiltration D. Calcification changes

B. Of the decrease of estrogen in post menopausal women Fibroids are estrogen dependent and will increase with pregnancy and decrease postmenopausally. The only sonographic difference between a leiomyoma and a leiomyosarcoma is a rapid increase in growth.

A rapidly growing fibroid in the first trimester of pregnancy is not as worrisome as a rapidly growing fibroid in a post menopausal woman because A. Of the rapid rise of progesterone in th e first trimester B. Of the decrease of estrogen in post menopausal women C. Rapid fibroid growth is not worrisome D. Both cases are cause for concern

C. Congenital mesoblastic nephroma Because of its vascularity, the fetus may experience heart failure and polyhydramnios. On ultrasound, a congenital mesoblastic nephroma will resemble a Wilm's tumor.

A rare renal tumor that is large, solid, and highly vascular is A. Neuroblastoma B. Renal teratoma C. Congenital mesoblastic nephroma D. None of the above

D. Distal acoustic shadows Distal acoustic shadowing is associated with a solid mass.

A simple cyst may exhibit all of the following except A. Anechoic interior B. Posterior enhancement C. Thin walled D. Distal acoustic shadows

D. Below the umbilicus The uterus has a dramatic decrease in size and returns to a position between the symphysis pubis and umbilicus.

A sonogram approximately 1 week after delivery would visualize the uterus A. Above the umbilicus B. 38 cm from the symphysis pubis C. 30 cm from the symphysis pubis D. Below the umbilicus

D. Hypertension Maternal hypertension can cause a restrictive flow in the uterine vessels, which in turn, may decrease the placental perfusion.

A thick placenta (>5cm) may be associated with all of the following except A. Gestational diabetes mellitus B. Infection C. Multiple gestations D. Hypertension

D. Triploidy Triploidy from the paternal component, presents with a large cystic placenta.

A thin placenta (<1.5cm) may be associated with all of the following except A. Preeclampsia B. IUGR C. Insulin-dependent diabetes mellitus D. Triploidy

D. A and B Transvaginal probes need to be cleaned with soap and water as well as soaked in a disinfecting solution. Transabdominal probes may be cleaned with a moist towelette.

AIUM guidelines for cleaning a transvaginal probe A. Clean with soap and water B. Immerse in disinfecting solution C. Clean with moist towelette D. A and B E. A and C F. All of the above

The premature separation of the placenta after 20 weeks of gestation.

Abruptio placentae

C. Hydranencephaly Hydranencephaly exists when the cerebral hemispheres are replaced by fluid. The brain stem is usually spared. Causes are thought to be infection or obstruction of the internal carotid artery.

Absent cerebral cortex is found in what cranial abnormality? A. Holoprosencephaly B. Hydrocephaly C. Hydranencephaly D. Agenesis of the corpus callosum

D. A and B Acrania is theorized to be the first trimester finding of anencephaly. With prolonged exposure to the amniotic fluid, the abnormal brain tissue is eroded, and the second trimester finding of anencephaly is appreciated.

Acrania is described as A. Abnormal brain tissue with absent calvarium B. The first stage of anencephaly before prolonged exposure to amniotic fluid C. Normal brain tissue with abnormal facies D. A and B E. All of the above

C. 18 weeks Development of the corpus callosum occurs between 12-18 weeks; therefore, visualization is not possible less than 18 weeks

Agenesis of the corpus callosum cannot be diagnosed by ultrasound before A. 10 weeks B. 14 weeks C. 18 weeks D. 28 weeks

B. False It is possible to have a normal face with holoprosencephaly, although the majority of cases will have facial anomalies.

All cases of alobar holoprosencephaly will demonstrate facial anomalies. A. True B. False

B. Caused by a second trimester obstruction. Multicystic dysplastic kidney disease is caused by a first trimester obstruction. The kidney is non functioning with ureteral atresia.

All of the following about multicystic dysplastic kidney disease is true except A. Caused by a first trimester obstruction B. Caused by a second trimester obstruction C. Multiple, varying sized cysts in the parenchyma D. Echogenic parenchyma

D. Types include: Fibromas, thecomas, Brenner tumors Sex cord-stromal tumors are: fibromas, thecoma, granulosa cell, and androblastoma (Sertoli-Leydig cell). They appear hypoechoic to echogenic with a mixed heterogeneous pattern and appear similar to each other on ultrasound.

All of the following about stromal tumors is correct except A. All have a similar sonographic appearance and can't be differentiated from one another. B. Solid, hypoechoic ovarian tumors C. Types include: fibromas, thecomas, Sertoli-Leydig cell tumors. D. Types include: fibromas, thecomas, Brenner tumors

C. Urethral stenosis Oligohydramnios can cause restrictive deformities such as clubfoot, hand posturing, and scoliosis due to restricted fetal movement. Oligohydramnios may also cause pulmonary hypoplasia. It will not cause urethral stenosis.

All of the following are complications of oligohydramnios except A. Clubfeet B. Hand posturing C. Urethral stenosis D. Pulmonary hypoplasia

D. CDE is based on the amplitude of the soundwave Blood flow direction and velocity are characteristic of color Doppler imaging.

All of the following characteristics of color Doppler energy are false except A. CDE can determine the direction of blood flow B. CDE can determine the velocity of blood flow C. The different colors represent flow toward or away from the transducer D. CDE is based on the amplitude of the soundwave

B. False Ventricular septal defects can be divided into the muscular and membranous regions. A muscular VSD is within the muscular portion of the septum and may be detected from a four-chamber view. A membranous VSD is just below the aortic valve and is confirmed in the long axis view of the heart.

All ventricular septal defects can be detected in the four-chamber view. A. True B. False

E. All of the above Amniocentesis can be used to test the level of fetal bilirubin, infection, and for a limited amount of short limb syndromes.

Amniocentesis may be used to test for A. Fetal infection B. Fetal bilirubin C. Specific short limb syndromes D. A and B E. All of the above

C. Amniotic bands can cause amputation or limb deformities. Amniotic band syndrome is a rupture of the amnion early in pregnancy. That rupture allows bands of amniotic tissue to float freely in the amniotic fluid. If these bands come in contact with the fetus, they can cause strictures, amputations, and adhesions to the band itself. Amniotic sheets, also known as uterine synechia, are caused by scars or adhesions in the uterus. The amnion and chorion grown around the synechia and form a thick membrane with two layers of amnion and chorion on either side of the membrane. It is attached to the uterus at both ends and does not impede the fetus in any way.

Amniotic bands differ from amniotic sheets because A. Amniotic bands are attached to the uterus at both ends B. Amniotic sheets often cause disruption in the first trimester C. Amniotic bands can cause amputation or limb deformities D. There is no difference between amniotic bands and sheets.

D. All of the above A paraovarian, or mesonephric cyst originates from the mesonephric duct. A paraovarian cyst may form, regardless of uterine or ovarian status. Ovarian tissue does occasionally remain after an oophorectomy, especially if adhesions were present. The remaining ovarian tissue can still function and produce a cyst. It is called ovarian remnant syndrome and should be considered with any cystic mass identified in a postoophorectomy patient.

An anechoic, smooth-walled cyst is identified in a patient who has had a hysterectomy/oophorectomy. The differential diagnosis should include A. Paraovarian cyst B. Ovarian cyst C. Mesonephric cyst D. All of the above

A. True The ovum is usually found on the side of the corpus luteum from which it was released. It is possible for the ovum to contralaterally implant one-third of the time.

An ectopic pregnancy will usually be on the same side as the corpus luteal cyst. A. True B. False

B. False The exact uterine volume cannot be determined by the sonogram.

An exact volume of amniotic fluid can be determined by measuring the four quadrants of the maternal abdomen. A. True B. False

A. Excessive maternal smoking Excessive maternal smoking has been linked to accelerated maturation of the placenta. It is not predictive, however, in actual placenta perfusion to the fetus.

An excessively accelerated calcific placenta in the second trimester is associated with A. Excessive maternal smoking B. Chromosomal abnormality C. Precursor to third trimester IUGR D. All of the above

B. Brain sparing effort An increased HC/AC ratio is the result of redistribution of fetal blood away from the bowel and directed to the fetal head.

An increased HC/AC is a suggestion of A. Late onset of IUGR B. Brain sparing effort C. Placental insufficiency D. All of the above

A. Trisomy 21 Trisomy 21 is associated with a decreased MSAFP and an increased hCG.

An increased MSAFP is associated with all of the following except A. Trisomy 21 B. Neural tube defect C. Maternal preeclampsia D. Gastroschisis

A. Anencephaly AFP is produced by the fetus. An abnormal concentration occurs whenever there is an abnormal opening in the fetus allowing an increased amount of the protein in the amniotic fluid.

An increased MSAFP3 may be associated with A. Anencephaly B. Spina bifida occulta C. Skin covered spina bifida D. A and C

D. All of the above Other causes for an increased fundal height include fibroids, incorrect dates, and molar pregnancy.

An increased fundal height may be caused by A. Macrosomic fetus B. Polyhydramnios C. Twins D. All of the above

D. Chorioadenoma destruens An invasive mole and a hydatidiform mole are excessive trophoblastic proliferation. Unlike the hydatidiform mole, chorioadenoma destruens is malignant and invades into the myometrium.

An invasive mole is also known as A. Hydatidiform mole B. Triploid molar pregnancy C. Endometrioma D. Chorioadenoma destruens

D. A and B Pentalogy of Cantrell involves defects of the lower sternum, diaphragm, diaphragmatic pericardium, abdominal wall, and intracardiac defects. It is associated with trisomy 13 and trisomy 18.

An omphalocele and ectopia cordis are identified on sonogram. What condition should be included in the differential? A. Pentalogy of Cantrell B. Trisomy 18 C. Beckwith-Wiedemann syndrome D. A and B E. All of the above

D. Endometrioma The internal component of an endometrioma is typically blood from bleeding ectopic endometrial tissue during menstruation. Differential diagnosis may include a dermoid tumor; however, most women tend to be asymptomatic with dermoids.

An ovarian mass identified on sonogram is complex, predominantly hypoechoic with septations. The patient complains of severe pain during menses. The most likely diagnosis is A. Corpus luteal cyst B. Granulosa cell C. Thecoma D. Endometrioma

E. All of the above When the increased AFP is unexplained, it is thought to be because of an increased placental transfer of AFP. The placental dysfunction can occur with various placental abnormalities that may cause third trimester complications.

An unexplained increased MSAFP3 can cause what third trimester complications? A. Premature rupture of membranes B. Placental abruption C. Preterm labor D. A and B E. All of the above

C. Normal renal pyramids Unless the kidney is echogenic or obstructed, the anechoic cysts in the periphery represent normal renal pyramids.

Anechoic cysts that surround the periphery of the kidney in a "string of pearls" appearance most likely represent A. Multicystic dysplastic kidney disease B. Autosomal dominant polycystic kidney disease C. Normal renal pyramids D. Early-stage obstructed renal pyramids

A. Stein-Leventhal syndrome Seroli-Leydig cell tumor is an androblastoma and Brenner's cell tumor is a transitional cell tumor. Chocolate cyst is another name for endometrioma

Another name for polycystic ovarian syndrome is A. Stein-Leventhal syndrome B. Seroli-Leydig cell C. Brenner's tumor D. Chocolate cyst

C. 50-70% Dandy-Walker malformation may occur as part of the mendelian disorder (abnormalities in multiple different organ systems) and is associated with chromosomal abnormalities.

Associated findings occur with Dandy-Walker malformation A. Rarely B. 10-25% of the time C. 50-70% of the time D. Always

C. 16 weeks Amnion begins to fuse with the chorion at 12 weeks and is routinely complete by 14-16 weeks.

At what gestational age should the amnion and chorion be fused? A. 8 weeks B. 10 weeks C. 16 weeks D. 20 weeks

F. All of the above Beckwith-Wiedemann syndrome is due to a dysfunction of the placenta excreting increased levels of growth hormone. This causes organomegaly, macroglossia, omphalocele, hemi hypertrophy, and cardiac abnormalities.

Beckwith-Wiedemann syndrome is a group of which of the following disorders? A. Omphalocele B. Shortening of limbs unilaterally C. Macroglossia D. Polyhydramnios E. A and C F. All of the above

F. All of the above Brachycephaly with a flat occiput is a feature of trisomy 21. Brachycephaly may also be present with a spina bifida due to ventricular enlargement. Most commonly, brachycephaly is a normal variant.

Brachycephaly is associated with what conditions? A. Trisomy 21 B. Normal variant C. Myelomeningocele D. A and B E. A and C F. All of the above

A. A large BPD and shorter OFD for gestational age Brachycephaly appears as a short, rounded head.

Brachycephaly will demonstrate A. A large BPD and shorter OFD for gestational age B. A small BPD and longer OFD for gestational age C. An HC <3SD of mean for GA D. An HC >2SD of mean for GA

D. A and B Although duodenal atresia has been detected in the first trimester, the classic "double bubble" sign is not usually present until the late second and early third trimesters.

By 12 weeks gestational age, the sonographer can identify what abnormalities? A. Conjoined twins B. Anencephaly C. Duodenal atresia D. A and B E. All of the above

C. 13 weeks After 13 weeks, the fetus is larger and it is technically more difficult to obtain the correct view to measure a CRL.

CRL is the appropriate dating measurement until what gestational age? A. 8 weeks B. 10 weeks C. 13 weeks D. 15 weeks

A. Insulin-dependent diabetes mellitus Caudal regression syndrome is associated with insulin-dependent diabetes mellitus in up to 16% of the cases. It is thought to occur with poor glucose control in the first trimester. Findings include sacral agenesis, spinal, and lower limb abnormalities, femoral hypoplasia, GI and GU abnormalities.

Caudal regression syndrome is associated with A. Insulin-dependent diabetes mellitus B. Chromosomal abnormalities C. Noonan's syndrome D. Hypertension

B. Hyperstimulation of the ovaries by infertility medications Risks for ovarian hyperstimulation syndrome include: advanced maternal age, polycystic ovarian disease, oral contraceptives, and obesity.

Causes for ovarian hyper stimulation may include A. Oral contraceptives B. Hyper stimulation of the ovaries by infertility medications C. Hydatidiform mole D. All of the above

D. All of the above IUGR may be found in chromosomal abnormalities, infection early in pregnancy, and placental insufficiency.

Causes of IUGR include A. Fetal infection B. Chromosomal abnormality C. Placental insufficiency D. All of the above

Single nostril nose

Cebocephaly

B. Trisomy 18 Isolated choroid plexus cyst has a small risk for trisomy 18. If other abnormalities are identified along with the choroid plexus cyst, the risk for trisomy 18 increases.

Choroid plexus cysts, when found with other associated abnormalities, have a strong association with a. Noonan's syndrome B. Trisomy 18 C. Trisomy 13 D. X-linked hydrocephaly

A. Hydramnios Complete PUV obstruction does not allow for any fetal urination, therefore severe oligohydramnios occurs.

Complete posterior urethral valve obstruction has all of the following ultrasound characteristics except A. Hydramnios B. Hydronephrosis C. Hydroureter D. "Keyhole" bladder E. Lax abdominal wall

E. All of the above Subnormal intelligence is reported in 40-70% of cases. Morbidity rates are 24%,but are improving with increased anesthesia and surgical techniques.

Complications associated with Dandy-Walker malformation include A. Chromosomal abnormalities B. Subnormal intelligence after birth C. Increased neonatal death D. A and B only E. All of the above

B. <20% difference in EFW between the twins Discordance in dichorionic/diamniotic twins is more acceptable because of their difference genetic makeup, provided that the smaller twin is not less than the 10th percentile in EFW. In monochorionic twins, EFW should be concordant and not differ by more than 20%.

Concordant growth in twins refers to A. <10% difference in EFW between the twins B. <20% difference in EFW between the twins C. Both fetuses within normal range on a normal growth curve D. Fetuses remaining at the same EFW percentile from exam to exam.

B. False Discordance in dichorionic/diamniotic twins is more acceptable because of their difference genetic makeup, provided that the smaller twin is not less than the 10th percentile in EFW. In monochorionic twins, EFW should be concordant and not differ by more than 20%.

Concordant twin growth is more important in dichorionic twins than monochorionic twins. A. True B. False

A. Twin pregnancy Twin pregnancy is associated with an elevated hCG.

Conditions associated with a poorly rising or decreasing hCG include all of the following except A. Twin pregnancy B. Ectopic pregnancy C. Anembrionic demise D. Incorrect dates

B. Paramesonephric ducts Congential abnormalities result from improper fusion of the mullerian, or paramesonephric ducts.

Congenital abnormalities of the uterus result from improper fusion of the A. Mesonephric ducts B. Paramesonephric ducts C. Gartner's duct D. Wolffian ducts

C. Macrocystic Macrocystic is defined as multiple large cysts measuring 2-10 cm.

Congenital cystic adenomatoid malformation is divided into three types. Type I is A. Microcystic B. Medium-sized cysts C. Macrocystic D. Mixed-size cysts

D. If liver is identified in the chest If the liver is intrathoracic, the survival rate is 43%, if the liver is intraabdominal, the survival rate is 80%.

Congenital diaphragmatic hernia has a poor prognosis if A. It is a left-sided defect B. The stomach is located on the left and anterior C. If bowel is identified in the chest D. If liver is identified in the chest

E. B and C Congenital hydrocephaly is an X-linked abnormality, with the expression in males and the females being carriers. It is able to be detected through DNA testing.

Congenital hydrocephalus is A. Genetically linked affecting both male and females B. Able to be detected in both male and females by DNA testing C. Expressed in males only D. A and C E. B and C F. All of the above

Premature fusion of cranial sutures resulting in abnormal shapes of the cranium.

Craniosynostosis

Absent nose with protrusion of tissue at leve of eye sockets.

Cyclopia

B. Abnormal facial findings Cyclopia - absent nose with protrusion of tissue at level of eye sockets; hypotelorism - close-set orbits; cebocephaly - single nostril nose; and cleft lip/palate are all abnormal facial findings strongly associated with holoprosencephaly.

Cyclopia, hypotelorism, proboscis, cebocephaly, and cleft lip/palate are A. Abnormal intracranial findings B. Abnormal facial findings C. Associated with hydrocephaly D. All of the above

B. False Seventy-five percent of cystic hygromas are associated chromosomal abnormalities. If the cystic hygroma is isolated, the outcome for the fetus is typically good.

Cystic hygromas are always associated with a chromosomal abnormality. A. True B. False

E. C and D Cystic hygromas are associated with elevated MSAFP and Turner syndrome.

Cystic hygromas are associated with A. Potter's syndrome B. Beckwith-Widemann syndrome C. Elevated levels of AFP D. Turner syndrome E. C and D F. B and D G. All of the above

A. Obstruction of the lymph system at the level of the jugular veins. A cystic hygroma occurs when the jugular lymph sacs fail to communicate with the venous system. The obstructed lymphatic fluid fills the sacs and forms the cystic hygroma.

Cystic hygromas are caused by A. Obstruction of the lymph system at the level of the jugular veins B. Obstruction of the lymph system at the level of the iliac veins C. Obstruction of the venous system at the level of the jugular veins D. Carotid artery obstruction

B. A cystic hygroma is a cystic mass separated by a midline septum. The midline septum represents the medial walls of the two obstructed lymph sacs.

Cystic hygromas differ from cervical masses because A. A cystic hygroma is hypoechoic with internal septations B. A cystic hygroma is a cystic mass separated by a midline septum C. A cystic hygroma appears as two completely separate sacs D. Cystic hygromas may not be differentiated from other cervical masses.

D. Vaginal agenesis Vaginal agenesis is the absence of a vagina. Gartner's duct cyst, Nabothian cyst, and hematocolpos will appear as a cystic mass in the vagina.

Cystic masses in the vagina could include all of the following except A. Gartner's duct cyst B. Nabothian cyst C. Hematocolpos D. Vaginal agenesis

D. A and B Dolicocephaly is associated with breech fetuses and oligohydramnios.

Dolicocephaly is often associated with what conditions? A. Breech fetus B. Oligohydramnios C. LGA fetus D. A and B E. A and C F. All of the above

B. A small BPD and longer OFD for gestational age Dolicocephaly appears as a long, narrow head.

Dolicocephaly will demonstrate A. A large BPD and shorter OFD for gestational age B. A small BPD and longer OFD for gestational age C. An HC <3SD of mean for GA D. An HC >2SD of mean for GA

C. Normal

Doppler sampling of the maternal uterine artery <26 weeks shows a diastolic notch. This notch is indicative of A. IUGR B. Maternal HTN C. Normal D. A and B

A. Umbilical cord An increase in systolic/diastolic ratio has been shown to have a correlation with placental insufficiency.

Doppler testing of vessels that may aid in the diagnosis of IUGR are A. Umbilical cord B. Straight sinus C. Celiac axis D. Jugular vein E. A and C F. All of the above

A. Trisomy 21 Thirty percent of fetuses with duodenal atresia have trisomy 21.

Duodenal atresia has a strong association with what chromosomal abnormality? A. Trisomy 21 B. Trisomy 18 C. Trisomy 13 D. Unbalanced translocation

C. The pelvis, moving superiorly to the renal fossa. Kidneys begin to develop at 7 weeks. Between 7 and 11 weeks, the kidneys migrate cephalically into the renal pelvis.

During organogenesis, the kidneys migrate from A. The thorax, moving inferiorly to the renal fossa B. The anterior midgut to the posterior renal fossa C. The pelvis, moving superiorly to the renal fossa D. The kidneys do not migrate

A. Thin, echogenic line, 4-8 mm The proliferative phase is day 5-9 post menstruation

During the early proliferative phase, the endometrium appears A. Thin, echogenic line, 4-8 mm B. Thin, hypoechoic line, 4-8 mm C. Thickened and hypoechoic medially with an echogenic basal layer D. Thickened and echogenic throughout

C. Thickened and hypoechoic medially with an echogenic basal layer The periovulatory, or late proliferative phase is day 10-14 postmentruation.

During the periovulatory phase, the endometrium appears A. Thin, echogenic line 4-8 mm B. Thin, hypoechoic line 4-8 mm C. Thickened and hypoechoic medially with an echogenic basal layer D. Thickened and echogenic throughout

D. Thickened and echogenic throughout The secretory phase is day 15-28 postmenstruation. The echogenicity is a result of edema of the functional zone of the endometrium.

During the secretory phase, the endometrium appears A. Thin, echogenic line 4-8 mm B. Thin, hypoechoic line 4-8 mm C. Thickened and hypoechoic medially with an echogenic basal layer D. Thickened and echogenic throughout

D. A and C Hypertension and maternal smoking can cause the placenta to undergo early maturation. Smoking can cause an increase in calcifications in the placenta. Unfortunately, placenta maturation has not proved to be a reliable tool in assessing placental function or fetal well-being.

Early placental maturation may be associated with A. Hypertension B. Gestational diabetes mellitus C. Maternal smoking D. A and C E. All of the above

C. The cervical region

Eighty percent of cystic hygromas occur in what region? A. The axilla B. The mediastinum C. The cervical region D. None of the above

C. Polyhydramnios Regardless of the presence of a TE fistula, 80% of fetuses derive polyhydramnios by the third trimester.

Eighty percent of fetuses with esophageal atresia have what associated condition in the third trimester? A. Intrauterine growth restriction B. Macrosomia C. Polyhydramnios D. Decreased abdominal circumference

A. Benign cystic teratoma An endometrioma may have a variety of appearances, but primarily appears as a cystic to hypoechoic mass with internal echoes or debris layering consistent with blood. Teratomas also have a variety of appearances, but can be hypoechoic with internal echoes and layering of debris.

Endometrioma may appear similar to A. Benign cystic teratoma B. Mucinous cystadenoma C. Thecoma D. Serous cystadenoma

A. Ectopic endometrial tissue The most common locations for endometriosis are: ovaries, uterine ligaments, rectovaginal septums, posterior cul-de-sac, and pelvic peritoneum.

Endometriosis is A. Ectopic endometrial tissue B. Benign invasion of endometrial tissue into the myometrium C. Endomyosarcoma with chocolate tissue D. Inflammation of the endometrium

B. Trisomy 18 Esophageal atresia is a very strong marker for trisomy 18.

Esophageal atresia has a strong association with what chromosomal abnormality? A. Trisomy 21 B. Trisomy 18 C. Trisomy 13 D. Translocation

D. Can be diagnosed in the first trimester Ninety percent of esophageal atresias have a tracheoesophageal fistula. This allows amniotic fluid to reach the stomach, but at a slower rate. The stomach will be visualized, but may be smaller than usual. Polyhydramnios occurs in the mid to late second trimester. The VACTERL complex is: Vertebral anomalies, Anal atresia, Cardiac abnormalities, Tracheoesophageal atresia, Renal anomalies, and Limb anomalies. At least three of the anomalies listed must be present to diagnose the VACTERL condition.

Esophageal atresia is A. Diagnosed by ultrasound 90% of the time. B. Associated with oligohydramnios C. Is a component of the VACTERL complex D. Can be diagnosed in the first trimester

B. False Ninety percent of esophageal atresias have a TE fistula that communicates with the stomach. Polyhydramnios may not develop until the third trimester, making a second trimester diagnosis difficult.

Esophageal atresia is routinely diagnosed by ultrasound because of an absent or small stomach bubble. A. True B. False

B. 30 seconds Fetal breathing must last at least 30 seconds in a 30 minute period in order to score a 2 in the BPP.

Fetal breathing must last how long to be counted in the BPP? A. 20 seconds B. 30 seconds C. 1 minute D. 2 minutes

E. All of the above Caudal regression syndrome and cardiac defects are first trimester insults and a risk of occurrence increases with increasing blood sugar levels in the first trimester. Shoulder dystocia can occur with delivery of macrosomic fetuses.

Fetal complications caused by maternal diabetes may include A. Caudal regression syndrome B. Cardiac defects C. Shoulder dystocia D. A and B E. All of the above

False Fetal demise is defined as an absence of fetal heart tones after 20 weeks.

Fetal demise is the detection of an absent heart beat after 16 weeks. True False

A. True Fibroids are estrogen dependent and will increased with pregnancy and decrease post menopausally.

Fibroids are estrogen-stimulated tumors. A. True B. False

D. A and B Thirty-three percent will have spinal defects, 36% will have cardiac defects

Fifty percent of fetuses with duodenal atresia also have which of the following abnormalities? A. Spinal defects B. Cardiac defects C. Macrosomia D. A and B E. All of the above

A. Dandy-Walker malformation A true Dandy-Walker malformation must have agenesis of the cerebellar vermis with communication to the fourth ventricle. A Dandy-Walker malformation variant is described as having some degree of cerebellar vermis agenesis, but not complete agenesis. An arachnoid cyst will push the cerebellum superiorly without splaying the cerebellum.

Findings on ultrasound include an increased cysterna magna, agenesis of the cerebellar vermis with communication to the fourth ventricle and ventriculomegaly. The most likely diagnosis would be A. Dandy-Walker malformation B. Dandy-Walker malformation variant C. Arachnoid cyst D. Communicating hydrocephaly

C. Arachnoid cyst An arachnoid cyst will not cause splaying of the cerebellum and the cerebellar vermis will be intact.

Findings on ultrasound include hydrocephaly, an enlarged cysterna magna, and an intact cerebellar vermis elevated by a cyst in the posterior fossa. The most likely diagnosis would be A. Dandy-Walker malformation B. Dandy-Walker malformation variant C. Arachnoid cyst D. Communicating hydrocephaly

C. Long axis view This view allows for visualization of the right ventricular outflow tract, the left ventricular outflow tract and the crossing of the outflow tracts.

From the four-chamber view, the transducer is tilted toward the fetal left shoulder. This will obtain what cardiac view? A. Five-chamber view B. Short axis view C. Long axis view D. Aortic arch

B. False Chromosomal and associated defects (non-gastrointestinal) with gastroschisis are very rare.

Gastroschisis has more associated defects and chromosomal abnormalities than omphaloceles. A. True B. False

A. Right side of the umbilical cord Although left-sided gastroschisis have been reported, it is typically to the right of the umbilical cord.

Gastroschisis is a break in the abdominal wall at the level of the umbilical cord. Where does this typically occur? A. Right side of the umbilical cord B. Left side of the umbilical cord C. At the umbilical cord insertion with cord attaching to the gastroschisis D. Medially and inferior to the umbilical cord

A woman who is pregnant

Gravida

D. All of the above Hemivertebrae is easiest to view in the sagittal plane of view because the other vertebrae may be used as a reference of normal. It is visible in the other planes of view also, but requires more meticulous scanning.

Hemivertebrae may be identified on sonogram A. As a narrowing of the individual vertebrae in the coronal plane of view B. As a narrowing of the individual vertebrae in the sagittal plane of view C. As a narrowing of the individual vertebrae in the axial plane of view D. All of the above

B. 14 weeks It may be possible to diagnose anencephaly as early as 12 weeks, but it should be definitely diagnosed by 14 weeks.

How early can anencephaly be detected on ultrasound? A. 10 weeks B. 14 weeks C. 16 weeks D. 20 weeks

D. A and B Fetal hydrops is defined as two sites of fluid accumulation or one site of fluid accumulation and fetal ascites.

How is the diagnosis of fetal hydrops made by sonography? A. Two fetal sites of fluid accumulation B. Fetal ascites and one site of accumulated fluid C. One site of fluid accumulation and oligohydramnios D. A and B E. A and C F. All of the above

A. 2-3 days Scalp edema can be seen 2-3 days, or 24-72 hours, after fetal death.

How long after fetal death can scalp edema be seen? A. 2-3 days B. 5-10 days C. 10-20 days D. 20-30 days

B. Every 30 minutes A normal fetal bladder should empty every 30-45 minutes.

How often does the fetal bladder fill? A. Every 10 minutes B. Every 30 minutes C. Every hour D. Every 2 hours

The cranial cavity is fluid filled and there is no remaining cortex. The falx cerebri and posterior fossa are normal.

Hydranencephaly

C. A vascular accident of the internal carotid arteries. It is a destructive process that obliterates the cerebral cortex. The brain stem is usually spared. Other causes include infection and intrauterine strangulation.

Hydranencephaly is thought to result from A. Chromosomal abnormalities B. A vascular accident of the jugular veins C. A vascular accident of the internal carotid arteries D. None of the above

A. Following the cystic spaces to ensure that they all communicate A hydrosalpinx may initially look like a cystic mass with septations. On closer examination, the septations are not complete. The sonographer is able to follow the connection of the cystic spaces. This assumes that the structure is tubular and communicates as with hydrosalpinx.

Hydrosalpinx may be differentiated from a multicystic ovarian mass on sonogram by A. Following the cystic spaces to ensure that they all communicate B. Using color Doppler to follow the ovarian artery to the ovary C. Having the patient roll to lengthen out the fallopian tube D. The two cannot be differentiated.

B. False In most cases, hypoplastic left heart syndrome (HLHS) is easily visualized by 18 weeks. It is possible for HLHS to evolve through the pregnancy due to aortic coarctation.

Hypoplastic left heart syndrome is easily apparent at 18 weeks. A. True B. False

Close set orbits

Hypotelorism

D. Spina bifida Spina bifida is not associated with IUGR. Diabetes with vasculopathy, maternal HTN, and chromosomal abnormalities may all be associated with IUGR.

IUGR can be caused by all except A. Diabetes B. Maternal hypertension C. Chromosomal abnormalities D. Spina bifida

A. EFW <10% Although the other features, such as oligohydramnios, do often coexist and interact with IUGR, the diagnosis of IUGR is a fetus <10% for gestational age.

IUGR is A. EFW <10% B. Decreased AFV C. Increased umbilical cord dopplers D. Abnormal growth ratios

C. Proboscis If the nose is not visualized, the sonographer should look superiorly on the fetal face for a proboscis. The orbits should also be identified, looking for hypotelorism or cyclopia.

If a fetal nose is not visualized, one should look for A. Cebocephaly B. Cyclopia C. Proboscis D. All of the above

C. 3/22/2002 Nagele's rule is: 1. identify the LMP 2. add 7 days 3. subtract 3 months 4. add one year

If a patient's LMP is 6/15/2001, by Nagele's rule, the EDC should be A. 2/22/2002 B. 2/30/2002 C. 3/22/2002 D. 3/30/2002

A. True Studies have shown a 22% rate of congenital abnormalities if hemoglobin (a reflection of blood sugar control) was >8.5% before 14 weeks. A 3% rate of congenital abnormalities was reported if hemoglobin was <8.5%.

If an insulin-dependent mother is in good diabetic control in the first trimester, the risk of structural abnormalities is decreased. A. True B. False

B. 50%

If one parent has autosomal dominant polycystic kidney disease, the risk for the fetus having the same disease is A. 25% B. 50% C. 75% D. 100%

B. Too high The scanning plane is too high. When performing a biparietal diameter measurement (BPD), the fetal head should be ovoid and the measurement obtained at the level of the thalami and cavum septum pellucidum.

If performing a BPD measurement and the midline echo is continuous and unbroken, this would indicate that the scanning plane is A. Normal B. Too high C. Through the fetal neck D. Correct

D. All of the above This condition is called situs inversus (partial or total), dextrocardia and heterotaxy syndrome. Along with dextrocardia, there may be significant intracardiac anomalies, anomalies of the great vessels, an interrupted IVC, an anomalous venous return system, asplenia or polysplenia, and possible heterotaxy of the abdominal organs depending on whether it is complete or partial.

If the axis of the heart is pointed to the right, the sonographer should look for A. An interrupted IVC B. Other heart abnormalities C. Abdominal organ orientation D. All of the above

B. Cloacal exstrophy Cloacal exstrophy is believed to arise from irregular development of the cloacal membrane, Neural tube defects are present 50% of the time.

If the bladder is herniated through the ventral wall and a spinal defect is identified, what condition should be considered? A. Bladder exstrophy B. Cloacal exstrophy C. Pentalogy of Cantrell D. Limb-body wall complex

B. Dolicocephaly Dolicocephaly is a long, narrow head with a small BPD when compared with a longer OFD.

If the cephalic index is <75, it is an indication for what condition? A. Brachycephaly B. Dolicocephaly C. Microcephaly D. Macrocephaly

A. Brachycephaly A cephalic index >85 describes an increased BPD when compared to a shorter OFD. The head has a rounded appearance on ultrasound.

If the cephalic index is >85, it is an indication for what condition? A. Brachycephaly B. Dolicocephaly C. Microcephaly D. Macrocephaly

C. Trisomy 18 If AFP (<0.6 MoM), uE3 (<0.5 MoM), and hCG (<0.3 Mom) are all decreased, the triple screen will show an increased risk for trisomy 18.

If the triple marker screen shows a decreased AFP, a decreased estriol, and a decreased hCG, the fetus is at risk for A. Trisomy 21 C. Smith-Lemli-Opitz syndrome C. Trisomy 18 D. Trisomy 13

B. False Both immune and nonimmune hydrops present the same sonographically.

Immune and nonimmune hydrops may be differentiated by their sonographic appearances. A. True B. False

D. Teardrop ventricles The "teardrop" appearance (enlargement of the atria and occipital horns and lateral displacement of the anterior horns) is present 90% of the time.

In 90% of cases with agenesis of the corpus callosum, what other sonographic finding is present? A. Polyhydramnios B. Omphalocele C. Polydactyly D. Teardrop ventricles

C. BPD, HC, AC, FL Multiple parameters help to increased the accuracy of estimated fetal weight.

In a fetus without abnormalities in the second and early third trimester, the best parameters to use for EFW are A. CRL B. FL/AC C. BPD, HC, AC, FL D. BPD/FL

A. True A cesarean section scar will appear at the cervicocorpus junction on the anterior myometrial wall.

In a nongravid uterus, a previous cesarean section scar may be visualized on sonogram. A. True B. False

A. An increased S/D ratio In response to hypoxia, the fetus reroutes blood to the brain in a brain-sparing effort. The middle cerebral artery is normally of higher resistance, but it will decrease to compensate for the increased blood flow and brain-sparing effort.

In a normal fetus, if the middle cerebral artery is sampled, one would expect to find A. An increased S/D ratio B. A decreased S/D ratio C. Retrograde flow D. Absent flow

B. False A membrane may be present with a dichorionic/diamniotic or monochorionic/diamniotic pregnancy. A zygocity indicator would be fetal genders (if different).

In a twin pregnancy with only one placenta, a dividing membrane is the only structure that needs to be identified to determine zygocity. A. True B. False

B. Meckel's diverticulum The yolk sac reduces in size as pregnancy advances. However, it may persist throughout pregnancy and continue to persist into adulthood. In about 2% of adults, the proximal intra-abdominal part of the yolk sac is presented as a diverticulum of the ilium, called Meckel's diverticulum.

In about 2% of adults, the yolk sac persists as a diverticulum of the ileum. This is known as A. Michael's diverticulum B. Meckel's diverticulum C. Turner's diverticulum D. Outside the chorionic cavity between the chorion and the endometrial wall

D. A and B The cardiac ventricular walls will thicken and contractility decreases. This causes the cardiac output to decrease. This may lead to acidosis, increased hematocrit, and increased neonatal morbidity.

In cases of fetal distress in diabetic mothers, the fetal heart may A. Have thickened ventricular walls B. Have decreased contractility C. Have increased cardiac output D. A and B E. All of the above

F. All of the above Partial situs inversus, divided into asplenia and polysplenia, has a 40% incidence of anomalies. They include complex heart disease, absent gallbladder, interrupted IVC with an azygous venous return and splenic abnormalities.

In cases of partial situs inversus, the sonographer should consider A. Cardiac defects B. Polysplenia C. Asplenia D. Interrupted IVC E. B and C F. All of the above

B. False The uterus atrophies equally in both multigravid and nulligravid women

In post menopausal women, the uterus wil not atrophy as much in multigrain women as nulligravid women A. True B. False

B. False The site and extension of the lesion may aid in a very general prognosis, but it is impossible to accurately and clearly predict an individual fetus' prognosis by in utero findings.

In the case of spina bifida, the fetal sonogram is able to accurately predict the child's impairment. A. True B. False

A. Enlarge In cases of a unilateral nonfunctioning kidney, the contralateral kidney will often enlarge to compensate. The unilateral kidney usually provides enough function to be sufficient for the individual.

In the case of unilateral, multicystic, dysplastic kidney disease, the contralateral kidney will often A. Enlarge B. Become obstructed C. Have decreased function D. Remain the same

D. All of the above CVS and PUBS may be performed provided the placenta or umbilical cord are accessible. If the fetal bladder is full, as in cases of PUV syndrome, fetal urine may be tested for karyotype.

In the cases of anhydramnios, how can karyotype be obtained? A. PUBS B. CVS C. Cystocentesis D. All of the above

D. 6 Three pairs of kidneys for in successive stages: pronephros, mesonephros, and metanephros with metanephros remaining as the functioning kidney.

In the early stages of organogenesis, how many kidneys form? A. 2 B. 3 C. 4 D. 6

C. Isoechoic to the fetal liver The fetal lung is isoechoic to the fetal liver in the second trimester

In the first and second trimesters, the fetal lung is A. Of greater echogenicity than the fetal liver B. Of decreased echogenicity compared to the fetal liver C. Isoechoic to the fetal liver D. Variable in appearance

A. The location of the fetal liver It is important to identify the location of the fetal liver. If the liver is intrathoracic, the prognosis for survival is 43%. If the liver is intrabdominal the prognosis is 80% survival.

In the identification of congenital diaphragmatic hernia, the sonographer should also assess A. The location of the fetal liver B. Fetal kidneys C. Fetal bladder D. All of the above

D. 70% of the time Due to the small mandible and normal sized tongue, normal swallowing is inhibited.

In the presence of micrognathia, polyhydramnios is present how often? A. 15% of the time B. 25% of the time C. 50% of the time D. 70% of the time

D. Less than 6 mm Although this is somewhat debated, many sources quote a number from 4-6 mm as the upper limit of normal in the second trimester.

In the second trimester, the normal renal pelvis size is A. Less than 20 mm B. Less than 15 mm C. Less than 8 mm D. Less than 6 mm

B. A single pocket of 1 cm cannot be measured This method is not as accurate as the amniotic fluid index.

In the single pocket method, when is the diagnosis of oligohydramnios made? A. A single pocket of 2 mm cannot be measured B. A single pocket of 1 cm cannot be measured C. A single pocket of 2 cm cannot be measured. D. A single pocket of 4 cm cannot be measured.

A. Of greater echogenicity than the fetal liver Although the lung increases in echogenicity throughout the pregnancy, researchers have not been able to correlate the increase with lung maturity.

In the third trimester, the fetal lung is A. Of greater echogenicity than the fetal liver B. Of decreased echogenicity compared tot he fetal liver C. Isoechoic to the fetal liver D. Variable in appearance

C. Ampulla The ovum enters the fallopian tube at the fimbriated ends. It courses to the ampulla where fertilization occurs 24-36 hours after ovulation.

In what portion of the fallopian tube does fertilization usually occur? A. Interstitial B. Isthmus C. Ampulla D. Infundibulum

D. None of the above Parvovirus, CMV, and HIV are all infections that may be transmitted across the placenta.

Infections that may affect the fetus include all of the following except A. Parvovirus B. Cytomegalovirus C. HIV D. None of the above

E. None of the above All of the above. They may all cause a complication in gastroschisis and should be monitored with ultrasound throughout the pregnancy.

Intrauterine complications of gastroschisis include all of the following except A. IUGR B. Edematous bowel wall C. Intestinal obstruction D. Dilated fetal stomach E. None of the above

B. False It is possible to ovulate during lactation. The risk of ovulation in the first 6 months while breast feeding is 1-5%.

It is not possible to get pregnant while lactating. A. True B. False

A. True In PUV cases, the fetal bladder may rupture creating urine ascites.

It is possible for the fetal bladder to rupture in utero. A. True B. False

B. Fetus >90% LGA refers to the fetus measuring greater than the 90th percentile for gestational age.

LGA is a term referring to A. A fetus weighing >4000g B. Fetus >90% C. A clinical assessment of an increased fundal height D. Polyhydramnios

C. Microcephaly MIcrocephaly is a result of a large portion of the brain tissue being herniated out of the cranium.

Large encephaloceles may be associated with A. Hydranencephaly B. Hypotelorism C. Microcephaly D. All of the above

B. False The presence of bowel without liver in the smaller omphaloceles has a stronger association to abnormal karyotypes and perinatal mortality.

Large omphaloceles have a worse prognosis than small omphaloceles. A. True B. False

1. Zygote 2. Cleavage 3. Morula 4. Blastocyst

List in sequence the embryologic stages following fertilization listed below: Morula Cleavage Zygote Blastocyst

D. A and B Depending on the fetal gestational age, lung maturity amniocentesis may be performed to assure lung maturation before delivery. in cases of complete PROM, often there is not an adequate sample of amniotic fluid available for maturity testing.

Lung maturity amniocentesis is used in what cases? A. IUGR fetus with decreasing EFW B. Insulin-dependent diabetes mellitus C. Complete PROM D. A and B E. All of the above

D. All of the above Pierre Robin syndrome, trisomy 18, and campomelic dysplasia are all associated with micrognathia and polyhydramnios. Micrognathia commonly causes difficulty swallowing resulting in polyhydramnios.

MIcrognathia may be associated with which of the following syndromes? A. Pierre Robin syndrome B. Trisomy 18 C. Campomelic dysplasia D. All of the above

B. Fasle Gastroschisis is a defect in the abdominal wall in which the bowel herniates into the amniotic fluid. This allows the AFP to enter the amniotic fluid directly from the abdominal wall defect, cross the placenta, and enter the maternal blood stream. The mean AFP value for gastroschisis is 7 MoM versus 4.1 MoM for omphaloceles.

MSAFP is elevated more with omphaloceles than with gastroschisis. A. True B. False

D. 97% of the time Other findings of Beckwith-Wiedemann syndrome include omphalocele, organomegaly, hemihypertrophy, and hypoglycemia.

Macroglossia is present how often in Beckwith-Wiedemann syndrome? A. 15% of the time B. 25% of the time C. 50% of the time D. 97% of the time

A. True Macroglossia may prevent effective fetal swallowing causing polyhydramnios.

Macroglossia may cause polyhydramnios. A. True B. False

A. Fetus weighing >4000g A fetus greater than the 90th percentile for estimated fetal weight is termed large for gestational age (LGA). A fetus greater than 4000g is macrosomic.

Macrosomia is A. Fetus weighing >4000g B. Fetus >90% C. Fetus with a shoulder thickness >10 mm D. LGA fetus

E. A and B Insulin-dependent diabetes mellitus (IDDM) and gestational diabetes mellitus (GDM) may both cause macrosomia. If the IDDM involves vascular disease, the fetus may actually be at risk for intrauterine growth restriction.

Macrosomia may be associated with A. IDDM B. GDM C. HTN D. Rh isoimmunization E. A and B

Prepubertal uterus - C. Corpus half the length of the cervix Adult nulliparous woman - B. Corpus and cervix equal in length Adult multiparous woman - A. Corpus twice the length of cervix

Match the following terms in column A with the definitions in column B: Column A Prepubertal uterus Adult nulliparous woman Adult multiparous woman Column B A. Corpus twice the length of cervix B. Corpus and cervix equal in length C. Corpus half the length of the cervix

CVS - B. Chorionic villi sampling can test for karyotype only. AFP is a fetal protein that may be tested for by concentration levels in the amniotic fluid or in maternal serum. Early amniocentesis - C. Early loss rate is thought to be associated with the unfused chorion and amnion. They are difficult to penetrate and often require multiple sticks. FISH - D. FISH is considered experimental and requires amniocentesis follow-up. Fluorescent chromosomal markers are introduced into the amniotic fluid that "tag" or attach to certain chromosomes. These markers and chromosomes fluorescence allow for early preliminary detection of chromosomal abnormalities. Amniocentesis - A. Amniocentesis may be performed after 14 weeks. It has a 0.3% risk of miscarriage. PUBS - E. PUBS - percutaneous umbilical blood sampling - is the insertion of a needle into the fetal umbilical cord and the withdrawal of fetal blood. It may be used in cases of anhydramnios, testing for isoimmunization, fetal blood typing, hemophilia, and other disorders.

Match the procedures in column A with the correct description in column B: Column A CVS Early amniocentesis FISH Amniocentesis PUBS Column B A. Amniotic fluid is withdrawn from the gestational sac. Performed after 14 weeks with a low loss rate. B. Performed 10-12 weeks. Cannot rule out spina bifida. C. Performed 11-14 weeks. Increased loss rate and associated with talipes equinovarus. D. "Tags" chromosomes 13, 18, 21, X, and Y for quick preliminary results. E. Technically difficult procedure that tests fetal blood.

Rhizomelia - B. - Shortening of the proximal limb. (Shortening of the humerus and femur.) Mesomelia - C. Shortening of the middle portion of the limb. (Shortening of the radius/ulna and tibia/fibula.) Micromelia - D. Shortening of all portions of a limb. (Shortening of the entire limb, but can also refer to shortening of a limb without specific reference.) Acromelia - A. Shortening of the hand and foot bones.

Match the term in column A with the definitions in column B: Column A Rhizomelia Mesomelia Micromelia Acromelia Column B A. Shortening of the hand and foot bones. B. Shortening of the proximal limb. C. Shortening of the middle portion of the limb. D. Shortening of all portions of a limb.

407 - D. Didelphic 408 - C. Unicornis 409 - A. Bicornuate 410 - B. Septate

Match the type of developmental anomalies of the uterus in this image with the terms in Column A Column A A. Bicornuate B. Septate C. Unicornis D. Didelphic

B. Fetal goiter Maternal Graves disease and Hashimoto thyroiditis produce antibodies that cross the placenta and may affect fetal thyroid production.

Maternal Grave's disease and Hashimoto thyroiditis may cause what finding in the fetus? A. Fetal ascites B. Fetal goiter C. Oligohydramnios D. There is no effect on the fetus

D. Gestational diabetes mellitus Gestational diabetes mellitus is a cause for macrosomia because of the increased maternal blood sugars.

Maternal causes of IUGR may include all of the following except A. Maternal infection B. Diabetic mothers with vasculopathy C. Maternal smoking D. Gestational diabetes mellitus

C. IUGR Hypertension can affect the vascular bed of the placenta resulting in intrauterine growth restriction.

Maternal hypertension may have what effect on pregnancy? A. Fetal abnormalities B. Chromosomal abnormalities C. IUGR D. Delayed lung maturation

F. B and D Maternally derived triploidy is associated with a small placenta. The fetus has severe asymmetric growth restriction and oligohydramnios.

Maternally derived triploidy has the following sonographic markers: A. Complete mole B. Severe assymetrical IUGR C. Large placenta with multiple cystic areas D. Oligohydramnios E. A and D F. B and D

D. A and B MIcrocephaly has been described as a HC between -2 and -3 standard deviations (SD) of the mean.

Microcephaly is defined as A. HC <2SD of mean B. HC >2SD of mean C. 2 week lag in HC D. A and B E. All of the above

C. Choroid is separated from the medial wall of the ventricle >5 mm MIld ventriculomegaly may be described as a separation of choroid from the medial ventricular wall >5 mm (but not dangling) or a measurement of the ventricular atrium between 10 and 15 mm.

Mild ventriculomegaly is described as A. Posterior horn measuring >15 mm B. Posterior horn measuring >5 mm C. Choroid is separated from the medial wall of the ventricle >5 mm D. Visualization of the third ventricle

D. All of the above Agenesis of the corpus callosum has mild ventriculomegaly, but the ventricle is also shaped like a "teardrop" in most cases. Mild ventriculomegaly has been associated with trisomy 21, but also be a normal variant.

Mild ventriculomegaly may be associated with A. Agenesis of the corpus callosum B. Trisomy 21 C. Normal D. All of the above

A woman who has been pregnant several times.

Multigravida

A woman who has given birth two or more times.

Multipara

B. Meninges and neural tissue Herniation of meninges alone is termed a meningocele.

Myelomeningocele refers to the herniation of A. Meninges B. Meninges and neural tissue C. Meninges at the lumbar level D. Meninges and neural tissue below L5

D. Chorioangioma Chorioangioma is a vascular mass arising from chorionic tissue and is similar to a hemangioma.

Name the placental vascular malformation that appears as a hypoechoic mass near the cord insertion. A. Fetal vascular anastomosis B. Placental lake C. Placental aneurysm D. Chorioangioma

A woman who has never been pregnant.

Nulligravida

A woman who has never given birth to a viable infant.

Nullipara

C. Maternal smoking Omphaloceles are not associated with teratogens, such as maternal smoking.

Omphaloceles are associated with all of the following except A. Advanced maternal age B. Chromosomal abnormalities C. Maternal smoking D. Cardiac abnormalities

E. All of the above Whenever the heart is deviated with the correct apex orientation, the sonographer should consider a thoracic mass.

On sonogram, the fetal heart is deviated to the right with apex pointed to the left. The sonographer should consider what possible abnormalities? A. Congenital diaphragmatic hernia B. Congenital cystic adenomatoid malformation C. Teratoma D. A and B E. All of the above

B. Unilateral UPJ obstruction A ureteropelvic junction (UPJ) obstruction is an obstruction at the junction of the ureter and renal pelvis. Therefore, the fetal urine is obstructed within the kidney causing hydronephrosis, but not hydroureter. As long as the contralateral kidney is functioning normally, the amniotic fluid should remain normal.

On sonogram, the fetus has unilateral hydronephrosis, nonvisualized ureters, normal bladder, and normal AFV. This most likely represents A. Unilateral UVJ obstruction B. Unilateral UPJ obstruction C. PUV D. None of the above

B. 5.5 weeks The yolk sac may be visible as early as 5 weeks, but is virtually always seen by 5.5 weeks.

On transvaginal sonography, when is the yolk sac visible? A. 4 weeks B. 5.5 weeks C. 6 weeks D. 7 weeks

A. Holoprosencephaly Dandy-Walker malformation is associated with other midline defects including agenesis of the corpus callosum and cephaloceles, as well as holoprosencephaly, clefting and cardiac defects. Dandy-Walker malformation has a 50-70% risk of associated abnormalities.

Other findings associated with Dandy-Walker malformation include A. Holoprosencephaly B. Facial clefting C. Cardiac defects D. All of the above

D. A and B Clubfeet and ventriculomegaly have a strong association with spinal defects.

Other findings associated with a spinal defect are A. Clubfeet B. Ventriculomegaly C. Single umbilical artery D. A and B E. All of the above

B. Endometroid Androblastoma, fibroma, thecoma, and granulosa cell are all sex cord-stromal tumors, with fribromas being the most common.

Ovarian tumors that appear similar to a fibroma include all of the following except A. Androblastoma B. Endometroid C. Thecoma D. All of the above

D. A and B Hyperstimulated ovaries produce large cysts that may gorse, as well as cause fullness and nausea to the patient. Rarely, more severe complications Can occur because of the shift in fluid resulting in ascites and effusions.

Ovaries that have been overstimulated by infertility medication are at risk for A. Producing grossly enlarged cysts B. Torsion C. Endometriosis D. A and B E. All of the above

F. All of the above

Overdistension of the urinary bladder may cause A. Anterior placenta to appear previa C. Closure of an incompetent cervix C. Distortion or closure of the gestational sac D. Obscured visualization of the internal iliac vein E. A and B only F. All of the above

C. On the 14th day of the menstrual cycle Ovulation occurs approximately 14 days after the first day of the last menstrual period.

Ovulation occurs approximately A. Always after intercourse B. On the 7th day of the menstrual cycle C. On the 14th day of the menstrual cycle D. On the 2nd day of the menstrual cycle

The number of pregnancies that have continued to viability.

Para

A. True Partial situs inversus, divided into asplenia and polysplenia, has a 40% incidence of anomalies. They include complex heart disease, absent gallbladder, interrupted IVC with an azygous venous return and splenic abnormalities.

Partial situs inversus is associated with more significant abnormalities than complete situs inversus. A. True B. False

D. A and C Parvovirus have also been referred to as 5th's disease and "slapped cheek" disease.

Parvovirus is also described as A. 5th's disease B. Chicken pox C. "Slapped cheek" disease D. A and C E. B and C

C. Large placenta with multiple cystic areas Paternally derived triploidy is associated with a relatively well-grown fetus that has a proportionate head size. The placenta is large with multiple cystic spaces resembling a molar pregnancy. This accounts for 90% of triploidy.

Paternally derived triploidy has the following sonographic markers: A. Complete mole B. Severe asymmetrical IUGR C. Large placenta with multiple cystic areas D. Oligohydramnios E. A and D F. B and D

B. False The physiological hernia is the outpouching of the intestines as they rotate around the SMA and then return to the abdomen.

Physiological herniation <12 weeks may also include bowel and liver. A. True B. False

Abnormal adherence of part or all of the placenta to the uterine wall.

Placenta accreta

Placental attachment to the myometrium without invasion

Placenta accreta vera

Invasion of placenta into the myometrium

Placenta increta

The abnormal adherence of part or all of the placenta in which the chorionic villi invade the myometrium.

Placenta increta

Invasion of placenta through the uterus and into other organs.

Placenta percreta

The abnormal adherence of part or all of the placenta in which the chorionic villi invade the uterine wall.

Placenta percreta

A. Invasion of placental tissue through the uterus into bladder Placenta accreta may be divided into 1. placenta accreta vera - placental attachment to the myometrium without invasion 2. placenta increta - invasion of placenta into the myometrium 3. placenta percreta - invasion of placenta through the uterus and into other organs.

Placenta percreta refers to A. Invasion of placental tissue through the uterus into bladder B. Invasion of placental tissue into myometrium C. Invasion of placenta up to the serosal layer D. Placental attachment to the myometrium without invasion

The implantation of the placenta in the lower uterine segment.

Placenta previa

An accessory lobe of placenta

Placenta succenturiata

C. The premature separation of placenta at any gestational age. Placental abruption usually occurs in the third trimester, although it is possible at any point in the pregnancy.

Placental abruption is A. The premature separation of placenta before 20 weeks B. The premature separation of placenta after 20 weeks C. The premature separation of placenta at any gestational age D. The same as a retrochorionic clot

E. All of the above Other risk factors include: abdominal trauma, smoking, sudden decompression of the uterus (ex: delivery of a twin or PROM), prolonged PROM, uterine anomaly, high pariety, and prior abruption.

Placental abruption is associated with A. Drug abuse B. Fibroids C. Maternal vascular disease D. A and C E. All of the above

B. False The diagnosis is determined by the history and physical findings. Most commonly, the patient presents with vaginal bleeding and abdominal or back pain.

Placental abruption is primarily diagnosed by ultrasound. A. True B. False

B. False Components of a BPP include flexion/extension, gross body movement, breathing, amniotic fluid, and NST.

Placental grading is an important component of BPP. A. True B. False

A. An increasing umbilical cord S/D ratio An increasing S/D ratio of the umbilical cord is a sign of vascular resistance within the placenta, which ultimately leads to a decrease in oxygen to the fetus.

Placental insufficiency is indirectly monitored by A. An increasing umbilical cord S/D ratio B. A decreasing umbilical cord S/D ratio C. Doppler of placental intervillous spaces D. Doppler of maternal arcuate arteries

C. Idiopathic Of the cases with polyhydramnios, 60% are idiopathic, 20% are structural, and 20% are maternal insulin dependent diabetes mellitus.

Polyhydramnios is most commonly associated with what finding? A. Insulin-dependent diabetes mellitus B. Duodenal atresia C. Idiopathic D. Micrognathia

F. All of the above Polyhydramnios has many causes that may include increased urine production or decreased fetal swallowing.

Polyhydramnios may be associated with which of the following? A. Osteogenesis imperfecta B. Cleft lip C. Maternal diabetes D. Twin-twin transfusion E. A and B F. All of the above

A. An extra digit on the ulnar aspect of the fetal hand Polydactyly may be isolated or occur as part of a syndrome. The extra digit may have a bone or be soft tissue only. Preaxial refers to the radial aspect the of the hand.

Postaxial polydactyl is A. An extra digit on the ulnar aspect of the fetal hand B. An extra digit on the radial aspect of the fetal hand C. Curvature of the last digit on the ulnar aspect D. An extra digit on the fetal foot

B. False Although it is overwhelmingly found in males, it may rarely be found in females. In females, it is usually caused by a cloacal malformation or urethral atresia.

Posterior urethral valve obstruction is found only in males. A. True B. False

A. Flattened facial features caused by the lack of amniotic fluid in PUV syndrome. Potter's fancies also includes low set ears, clubfoot, and hip dislocation.

Potter's fancies refers to A. Flattened facial features caused by the lack of amniotic fluid in PUV syndrome. B. Abnormal development of the abdominal muscles caused by the overdistended bladder in PUV syndrome. C. The appearance of the fetal bladder in PUV D. Other name for urinary ascites.

B. An extra digit on the radial aspect of the fetal hand. Polydactyly may be isolated or occur as part of a syndrome. The extra digit may have a bone or be soft tissue only. Postaxial refers to the ulnar aspect of the hand.

Preaxial polydactyl is A. An extra digit on the ulnar aspect of the fetal hand. B. An extra digit on the radial aspect of the fetal hand. C. Curvature of the last digit on the ulnar aspect. D. An extra digit on the fetal foot.

A woman who is pregnant for the first time.

Primigravida

A woman who has given birth one time to a viable infant, regardless of whether the child was living at birth and regardless of whether the birth was single or multiple.

Primipara

D. Beginning with the expulsion of the placenta The puerperium period begins with the expulsion of the placenta and continues until maternal physiology and anatomy return to a prepregnancy level, approximately 6-8 weeks.

Puerperium refers to the period A. Surrounding conception time B. After death C. 6-8 weeks before delivery D. Beginning with the expulsion of the placenta

A. True Approximately 10-15% of pulmonary sequestrations are found within or below the diaphragm, typically on the left side.

Pulmonary sequestrations may be found in the fetal abdomen. A. True B. False

C. 1,3,2,4 The occurrence rates for ectopic locations are: Ampullary - most common, 75-80% Isthmus - second most common, 10-15% Ovarian - 0.5% Cervical - 0.1%

Rank the following ectopic pregnancy implantation sites from the most common to the least common. 1 - Ampullary 2 - Ovarian 3 - Isthmus 4 - Cervical A. 1,2,3,4 B. 4,2,3,1 C. 1,3,2,4 D. 3,1,4,2

B - fetal tone (the central nervous center that regulates fetal tone functions first at 7.5 - 8.5 weeks.) A - Body motion (The central nervous center that regulates body movements starts functioning at 9 weeks) C - Breathing (The central nervous center that regulates fetal breathing starts at 21 weeks.) D - fetal heart rate acceleration (The central nervous center for fetal heart rate reactivity functions by the end of the second trimester or beginning of the third trimester.

Rank the following in order of their neurological development, earliest to latest: A. Body motion B. Fetal tone C. Breathing D. Fetal heart rate acceleration

No dilation

Renal Grade 0

Renal pelvic dilation with or without infundibula visible

Renal Grade 1

Renal pelvic dilation with calices visible.

Renal Grade 2

Renal pelvis and calices dilated.

Renal Grade 3

Renal pelvis and calices dilated with parenchyma thinning.

Renal Grade 4

D. Mother Rh- and fetus Rh+ If a mother is Rh-, she will produce antibodies against an Rh+ fetus. The mother's antibodies will perceive the fetal blood as foreign and attack the fetal red blood cells resulting in erythroblastosis fetalis.

Rh isoimmunization results from which of the following combinations? A. Mother Rh- and father Rh- B. Mother Rh+ and father Rh- C. Mother Rh+ and fetus Rh- D. Mother Rh- and fetus Rh+

C. Previous cesarean section Implantation sites at risk for placenta accreta are uterine scars, submucosal fibroids, lower uterine segment, rudimentary horn, and uterine cornua.

Risks for placenta accreta include A. Maternal hypertension B. Isoimmunization C. Previous cesarean section D. Infertility

D. All of the above In the case of IUGR, the fetus will direct more blood flow to the brain. This will lower the PI and S/D ratio of the middle cerebral artery (MCA). More recently, multiple studies have been conducted showing that the peak velocity of the MCA is a good predictor of fetal anemia. The peak velocity is measured and plotted of a curve to determine whether the fetus is in need of a fetal blood transfusion because of fetal anemia.

Sampling of the middle cerebral artery is helpful in determining A. IUGR and related complications B. Prediction of fetal anemia C. The need for a fetal transfusion D. All of the above

D. All of the above Other signs of heart failure include pericardial effusions, decreased contractility, increased ventricular thickness, abnormal umbilical cord, and middle cerebral artery Doppler.

Sign of congestive heart failure in the fetus are A. Serous effusions B. Enlarged fetal liver C. Fetal ascites D. All of the above

C. Low amniotic fluid In cases of renal agenesis, particularly after 16 weeks, anhydramnios is present.

Sonographic features of renal agenesis include all except A. Inability to demonstrate blood flow in renal arteries. B. Elongated adrenal glands in the renal fossa C. Low amniotic fluid D. Unable to visualize fetal bladder

D. Cystic hygroma Cystic hygroma is associated with Turner syndrome, not Trisomy 18.

Sonographic findings with Trisomy 18 include all of the following except: A. IUGR B. Clenched hands C. Holoprosencephaly D. Cystic hygroma

D. AFV Although an increased AFV is associated with macrosomia, it is not a direct assessment of macrosomia.

Sonographic markers that may be used to assess macrosomia include all of the following except A. Humeral shoulder thickness B. Cheek-to-cheek diameter C. HC/AC ratio D. AFV

D. Oligohydramnios When the bowel perforates, meconium enters the peritoneal space. A membrane forms that seals off the intestine at the site of the perforation. The meconium that entered the peritoneum may cause calcium deposits. Other findings are polyhydramnios, ascites with echogenic debris, and bowel dilation.

Sonographic signs of bowel perforation include all of the following except A. Thickened bowel loops B. Abdominal calcifications C. Meconium cysts D. Oligohydramnios

D. A and C The hypoechoic region behind the placenta should measure 1-2 cm in thickness. Any increase in thickness of this area should alert the sonographer to a possible hematoma.

Sonographic signs of placental abruption include A. Retroplacental veins >2 cm B. Echogenic periplacental hematomas C. Hypoechoic periplacental hematomas D. A and C E. All of the above

B. False Spina bifida occulta is a mild form of spina bifida that does cause a break in the skin. Because the spinal cord does not herniate out of the spinal canal, it does not cause a lemon or banana sign.

Spina bifida occulta may be diagnosed on ultrasound by then lemon and banana sign. A. True B. False

C. Also known as subchorionic fibrin deposition Placental lakes are areas of fibrin under the chorion, on the fetal side of the placenta. They carry no clinical significance.

Subchorionic placental lakes are A. Associated with IUGR B. Associated with oligohydramnios C. Also known as subchorionic fibrin deposition D. All of the above

D. All of them Teratomas can occur in many different locations. The most common region is sacrococcygeal accounting for 50% of fetal teratomas. The second most common location is orofacial (including intracranial) and cervical, accounting for 5% of fetal teratomas.

Teratomas in pregnancy are located in A. The sacrococcygeal region B. Intracranial C. Cervical D. All of them

A. True The incidence of abnormalities in a breech fetus is 6.3% versus 2.4% in a vertex fetus.

Term breech fetuses are more likely to have abnormalities than vertex fetuses. A. True B. False

D. 99% of the time

The "banana" sign is present with spinal defects A. 50% of the time B. 75% of the time C. 85% of the time D. 99% of the time

C. The appearance of the cerebellum in the presence of a spinal defect. The "banana" sign refers to the displacement of the cerebellum inferiorly into the upper cervical canal. On transverse view, the cerebellum is small and resembles a banana.

The "banana" sign of the fetal cranium in diagnosing spina bifida refers to A. The narrowing of the vertebral process at the area of the defect B. The overall appearance of the fetal spine in the presence of a defect. C. The appearance of the cerebellum in the presence of a spinal defect D. The appearance of the fetal skull in the presence of a spinal defect

B. Duodenal atresia Amniotic fluid fills the stomach and duodenum proximal to the site of obstruction. The ultrasound appearance resembles a "double bubble."

The "double bubble" sign refers to A. Two-vessel umbilical cord B. Duodenal atresia C. Ureterocele D. Hypotelorism

D. The appearance of the fetal skull in the presence of a spinal defect. The "lemon" sign refers to the narrowing of the parietal bones giving the appearance of a lemon-shaped cranium in the axial view.

The "lemon" sign of the fetal cranium in diagnosing spina bifida refers to A. The narrowing of the vertebral process at the area of the defect B. The overall appearance of the fetal spine in the presence of a defect C. The appearance of the cerebellum in the presence of a spinal defect D. The appearance of the fetal skull in the presence of a spinal defect

D. B and C The twin peak is formed when the placental tissue migrates between the chorionic layers. This is 94-100% predictive of dizygotic twins.

The "twin peak" sign is: A. Also known as the beta sign B. A triangular projection of chorion into the dividing membrane. C. A sonographic predictor for dizygotic twins D. B and C E. All of the above

B. False There are many other antibodies in addition to the anti-D antibody that can cause immune hydrops.

The Rh immune globulin is effective against the anti-D antibody; therefore, immune hydrops no longer exists in anyone able to receive r hogan. A. True B. False

B. False The data for transverse cerebellar diameter is based on imaging in the axial plane of view.

The TCD may be measured in either the axial or coronal view. A. True B. False

C. Bulging bag of water BBOW, bulging bag of water, refers to the amniotic membrane bulging into the vagina.

The abbreviated BBOW refers to A. Baby below outer water B. Brachycephalic baby on way C. Bulging bag of water D. Baby below occipital wing

D. Premature rupture of membranes D. PROM, premature rupture of membranes, is the rupture of membranes before 37 weeks.

The abbreviation PROM refers to a. Partially ruptured outer membrane B. Previous rip of mucus plug c. Partial range of motion D. Premature rupture of membranes

C. Vaginal birth after cesarean

The abbreviation VBAC refers to A. Vaginal blockage and closure B. Vaginal blockage after cesarean C. Vaginal birth after cesarean D. None of the above

B. Hydrops fetalis Hydrops fetalis is an accumulation of fluid in body cavities and soft tissue. This image shows abdominal ascites, pleural effusion and anasarca.

The abnormality in this image is: A. Abdominal ascites B. Hydrops fetalis C. Anasarca D. Prune-belly syndrome

B. +/- 10 days Fetal growth is starting to show variation and multiple parameters are used to calculate the EDC. Both of these factors allow for an increased range of error.

The accuracy of gestational age from 13-20 weeks is A. 3-5 days B. +/- 10 days C. +/- 14 days D. +/- 21 days

C. +/- 14 days Fetal growth is showing a moderate amount of variation, which allows for an increasing range of error.

The accuracy of gestational age from 20-30 weeks is A. 3-5 days B. +/- 10 days C. +/- 14 days D. +/- 21 days

A. 3-5 days Fetal growth in the first trimester is very uniform, thus allowing for accurate dating.

The accuracy of gestational age in the first trimester is A. 3-5 days B. +/- 10 days C. +/- 14 days D. +/- 21 days

D. +/- 21 days Fetal growth has a large amount of variation in the third trimester and obtaining the images for EFW can be challenging depending on fetal position and size. This allows for the largest range of error in the pregnancy.

The accuracy of gestational age in the third trimester is A. 3-4 days B. +/- 10 days C. +/- 14 days D. +/- 21 days

D. When the amniotic fluid index is less than the 2.5th percentile All four quadrants are added and compared to an expected amniotic fluid volume for that fetus' gestational age. The normal range extends from 2.5% to 97.5%

The amniotic fluid index method of measuring four quadrants, the diagnosis of oligohydramnios is made A. When the amniotic volume is less than 300mL B. When the amniotic volume is less than 200mL C. When the amniotic fluid index is less than the 10th percentile D. When the amniotic fluid index is less than the 2.5th percentile

C. 16 weeks Although the kidneys produce some urine prior to 16 weeks, they do not take over the majority of urine production until after 16 weeks.

The amniotic fluid production is performed primarily by the fetus after what gestational age in pregnancy? A. 8 weeks B. 10 weeks C. 16 weeks D. 25 weeks

C. Proliferative phase In the proliferative phase, the lining is thick, but the internal component is hypoechoic. This allows for the echogenic polyp to be seen. In the secretory phase, the entire endometrial lining is echogenic and will mask a polyp.

The best time to visualize a polyp is during what stage of the menstrual cycle? A. Menstruation phase B. Follicular phase C. Proliferative phase D. Secretory phase

D. Tetralogy of Fallot The most predictive sonographic findings are the overriding aorta and VSD. If the VSD is perimembranous, it will not appear on the four-chamber view. The right ventricle may appear larger than the left, but this is not a consistent finding and is dependent on the degree of pulmonary stenosis. Differential diagnosis would include truncus arteriosis.

The cardiac abnormality consisting of a VSD, an overriding aorta, a small or atretic pulmonary trunk, and right ventricular hypertrophy describes A. Double outlet right ventricle B. Hypoplastic left heart syndrome C. Transposition of the great vessels D. Tetralogy of Fallot

D. Left ventricular aneurysm The image represents a left ventricular aneurysm. Notice the normal sized right ventricle and atria, as well as the normally positioned valves. The coexistent normal findings would rule out hypoplasia and Ebstein's anomaly.

The cardiac abnormality demonstrated in this image is: A. Hypoplastic left heart B. Hypoplastic right heart C. Ebstein's anomaly D. Left ventricular aneurysm

A. Fetal component of the placenta

The chorion frondosum progressively develops to become A. Fetal component of the placenta B. Maternal component of the placenta C. The amniotic cavity D. The yolk sac and stalk

D. AFV, gross body movement, fetal flexion/extension, fetal breathing, and NST. If the component (ex: movement) is present, it is given a score of two. If it is absent, it is given a zero with the total score being 10.

The components of BPP are A. Fetal breathing, Doppler, NST, gross body movement, and AFV B. Placental grade, NST, gross body movement, and AFV C. NST, Doppler, gross body movement, AFV, and fetal flexion/extension D. AFV, gross body movement, fetal flexion/extension, fetal breathing, and NST.

C. Theca lutein Theca lutein cysts are present in 18-37% of hydatidiform moles.

The cyst commonly associated with hydatidiform mole is A. Follicular B. Paraovarian C. Theca lutein D. Corpus luteal

D. The measurement is >10 mm

The cysterna magna is considered increased when A. The measurement is >5 mm B. The cerebellum may be seen outlined by fluid C. The cerebellar vermis is splayed D. The measurement is >10 mm

D. The date when menstrual bleeding began LMP refers to the first day of the last cycle.

The date of the last menstrual period indicates A. The date when fertilization occurred B. The date when menstrual bleeding ended C. The date when ovulation occurred D. the date when menstrual bleeding began

B. The maternal component of the placenta

The decidua basalis progressively develops to become A. Fetal component of the placenta B. Maternal component of the placenta C. The amniotic cavity D. The yolk sac and stalk

B. Placental edge <2 cm from internal os If the placenta is greater than 2 cm from the internal os, a vaginal delivery is considered safe

The definition of "low lying placenta" in the third trimester is A. Placental edge >3 cm from internal os B. Placental edge <2 cm from internal os C. Placental edge <3 cm from internal os D. Placental edge in lower uterine segment

C. CI = BPD/OFD x 100 A change in head shape, such as brachycephaly or dolicocephaly, affects accurate measurement in predicting gestational age. The degree to which fetal head shape affects BPD can be estimated with the formula: CI = BPD/OFD x 100

The degree to which the head shapes may affect biparietal diameter (BPD) can be estimated by which of the following formulas? A. HC = BPD2 + OFD B. V = 7(pi/6(BPD/OFD) C. CI = BPD/OFD x 100 D. BPD = BPD x OFD/1.256

D. Transvaginally There is no contraindication to scanning a placenta previa transvaginally, and it provides the most accurate diagnosis. Transabdominal scanning may give a false-positive caused by a full maternal bladder compressing the internal os or an inadequate view of the internal os.

The diagnosis of placenta previa is most accurately made A. Transabdominally with a full maternal bladder B. Transabdominally with an empty maternal bladder C. Transrectally D. Transvaginally

A. Greater than 10 mm in the posterior horn In the absence of a spinal defect, if the lateral ventricles measure greater than 10 mm, it is commonly associated with an obstruction of the ventricular system.

The diagnosis of ventriculomegaly may be made when the ventricle measures A. Greater than 10 mm in the posterior horn B. Greater than 15 mm in the posterior horn C. When the third ventricle may be visualized D. When the choroid does not touch the medial wall of the lateral ventricle

C. Corpus luteal cyst A corpus luteal cyst and ectopic pregnancy may mimic each other, and the sonogram should be correlated with clinical findings.

The differential diagnosis for a thick-walled ovarian cyst with blood flow surrounding the periphery is A. Follicular cyst B. Oocyte C. Corpus luteal cyst D. All of the above

E. All of the above Alobar holoprosencephaly and severe hydrocephaly may sonographically present similar to hydranencephaly. Lobar holoprosencephaly sonographically presents with a interhemispheric fissure anterior and posterior, thus being excluded in the differential.

The differential diagnosis for hydranencephaly may be A. Semilobar holoprosencephaly B. Alobar holoprosencephaly C. Severe hydrocephaly D. A and C E. B and C F. All of the above

E. A and C This mass may be either a hemorrhagic corpus luteal cyst or a mucinous cystadenoma. The patient's history, age, and stage in menstrual cycle may help to further differentiate the mass.

The differential diagnosis for this ovarian image is: A. Mucinous cystadenoma B. Dermoid C. Hemorrhagic corpus luteal cyst D. Tubo-ovarian abscess E. A and C F. All of the above

D. Arnold-Chiari malformation Arnold-Chiari malformation is most commonly associated with myelomeningocele and hydrocephaly.

The downward placement of the cerebellar vermis, the fourth ventricle, and the medulla oblongata through the foramen magna is termed. A. "lemon" sign B. Dandy-Walker malformation C. Arachnoid cyst D. Arnold-Chiari malformation

A. 4 weeks A gestational sac can be seen by transvaginal ultrasound as early as 4 weeks.

The earliest age at which a gestational sac may be visualized by transvaginal sonography is A. 4 weeks B. 6 weeks C. 8 weeks D. 10 weeks

C. 6 mm Normal endometrial lining in postmenopausal women not on hormone replacement therapy is <5-6 mm.

The endometrial lining in postmenopausal women not on hormone replacement therapy should be less than A. 10 mm B. 8 mm C. 6 mm D. 2 mm

B. 8 mm Normal endometrial lining in postmenopausal women on hormone replacement therapy is <8 mm.

The endometrial lining in postmenopausal women on hormone replacement therapy should be less than A. 10 mm B. 8 mm C. 6 mm D. 2 mm

D. All of the above Decidual proliferation in ectopic pregnancy and endometrial carcinoma can also cause the endometrium to appear echogenic.

The endometrium can appear echogenic 1. in the secretory phase 2. in patients with pelvic inflammatory disease 3. after dilation and curettage 4. in patients with endometritis 5. after removal of an intrauterine contraceptive device A. 1 and 3 only B. 4 only C. 1, 2, 3, and 5 D. All of the above

C. Fertilization of an ovum without any active chromosomal material

The etiology of hydatidiform mole is A. Trophoblastic changes in a blighted ovum B. Hydatid swelling of the retained placenta in a missed abortion C. Fertilization of an ovum without any active chromosomal material D. A and B

D. 32 weeks Visualization of the femoral epiphyseal plate is seen on fetuses with a gestational age greater than 33 weeks with 95% accuracy.

The femoral epiphyseal plates are visualized on sonogram after A. 13 weeks B. 20 weeks C. 24 weeks D. 32 weeks

C. Large liver The horizontal position of the fetal heart is largely due to a large liver size.

The fetal heart is horizontal in the chest because of A. Large spleen B. Flat diaphragm C. Large liver D. Large thorax

C. Foramen ovale The foramen ovale allows for oxygenated blood to pass from the right atrium to the left atrium.

The fetal shunt between the left and right atria is A. Ductus venosum B. Ductus arteriosis C. Foramen ovale D. Pulmonary ductus

B. Ductus arteriosis The ductus arteriosis allows for approximately 70% of the blood to bypass the nonfunctioning lungs.

The fetal shunt connecting the transverse aortic trunk and the main pulmonary trunk is A. Ductus venosum B. Ductus arteriosis C. Foramen ovale D. Pulmonary ductus

B. False The umbilical vein carries oxygenated blood

The fetal umbilical vein carries deoxygenated blood. A. True B. False

B. 13 weeks Fetal urine production begins at 12 weeks, but the fetal kidneys do not produce the majority of the fetal urine until 16 weeks.

The fetus begins to produce urine at what gestational age? A. 10 weeks B. 13 weeks C. 17 weeks D. 20 weeks

A. Anterior The fetus would be lying on its right side, with the left side closest to the maternal abdominal wall.

The fetus is breech with the spine on the maternal left. The fetal left side should be A. Anterior B. Posterior C. Inferior D. Superior

A. Anterior The fetus would be lying on its right side, with the left side closest to the maternal abdominal wall.

The fetus is transverse with head on the maternal left. The fetal spine is inferior. The fetal left side should be A. Anterior B. Posterior C. Inferior D. Superior

A. Anterior The fetus would be lying on its right side, with left side closest to the maternal abdominal wall.

The fetus is vertex with the spine on the maternal right. The fetal left side should be A. Anterior B. Posterior C. Inferior D. Superior

A. True If the diabetic mother has significant vascular disease as a component of her diabetes, it may cause intrauterine growth retardation in the fetus.

The fetus of an insulin-dependent diabetic mother may suffer from IUGR. A. True B. False

E. All of the above An increased nuchal sonolucency is associated with specific chromosomal abnormalities, genetic syndromes, and structural defects.

The findings in this image are associated with: A. Cardiac defects B. Skeletal dysplasia C. Down syndrome D. A and C E. All of the above

B. Yolk sac The first finding is a gestational sac, however ectopic pregnancies may also present with a pseudosac. A yolk sac is the earliest definitive sonographic sign of an intrauterine pregnancy.

The first definitive sonographic sign of an intrauterine pregnancy is A. Gestational sac B. Yolk sac C. Fetal pole D. Double decidua sign

D. Lymphatic fluid The fluid within a cystic hygroma is lymphatic fluid from an obstructed lymph system.

The fluid within a cystic hygroma is A. Serous fluid B. Amniotic fluid C. Ascites D. Lymphatic fluid

A. The largest abdominal tumor A mucinous cystadenoma is the largest ovarian tumor.

The following are true characteristics of a mucinous cystadenoma except A. The largest abdominal tumor B. Thick septations C. Benign D. May have debris layering within cyst

A. Abbreviation for hemolysis, elevated liver enzymes, and low platelets. Sonographic fetal HELLP findings include IUGR, oligohydramnios, and possible signs of fetal distress (poor BPP, abnormal UC Dopplers for example).

The following information about HELLP syndrome is true except A. Abbreviation for hemolysis, elevated liver enzymes, and low platelets B. Abbreviation for hemivertebrae, elevated liver enzymes, and low lying placenta C. Treated similar to severe preeclampsia D. Doppler may help in assessing HELLP syndrome

C. Largest EFW - smallest EFW / largest EFW x 100 Twin fetal discordance is determined by subtracting the largest twin from the smallest twin and dividing by the largest twin. This number is multiplied by 100 to determine the percentage.

The formula for determining EFW discordance is? A. Smallest EFW + largest EFW/smallest EFW B. Smallest EFW x 2 - largest EFW/largest EFW C. Largest EFW - smallest EFW/largest EFW x 100 D. Largest EFW x 100 - smallest EFW/largest EFW

A. d1 x d2 x d3 x .523

The formula for determining ovarian volume is A. d1 x d2 x d3 x .523 B. d1 + d2 + d3 x 3.14 C. d1 x d2 x d3 x 3.14 D. d1 x d2/d1 + d2 + d3

D. All of the above The functions of the yolk sac are: nutrition-transfer of nutrients to the embryo; hemopoiesis-blood cell development; and development of sex cells that later become spermatogonia or oogonia.

The functions of the secondary yolk sac are which of the following A. Nutrients for the embryo B. Hematopoiesis C. Contributing to the development of the reproductive system. D. All of the above

B. 30% The ratio of the heart circumference to the thoracic circumference should be 30-50%.

The heart should occupy what percentage of the fetal thorax? A. 25% B. 30% C. 60% D. 75%

B. Erythroblastosis fetalis This condition will cause immune hydrops in the fetus.

The hemolytic process of destruction of fetal red blood cells by the maternal antibodies is termed A. Hypercoagulation B. Erythroblastosis fetalis C. Thrombocytopenia D. Nonimmune hydrops

C. Paramesonephric cyst Hydatid of Morgagni is the most common paramesonephric cyst. It measures 2-10 cmm and appears similar to ovarian cysts.

The hydatid of Morgagni is A. A complex ovarian mass B. A section of sigmoid colon visible on tranvaginal ultrasound C. Paramesonephric cyst D. Another name for ovarian remnant syndrome

D. Dandy-Walker malformation The image is a Dandy-Walker malformation. The cerebellum is splayed, and the vermis is absent. There is communication with the fourth ventricle.

The image represents A. Arachnoid cyst B. Communicating hydrocephaly C. "Banana" sign D. Dandy-Walker malformation

B. 1/3 Although kidneys grow throughout gestation, the ratio of kidneys to abdomen remains constant at 0.27-0.30.

The kidneys should occupy how much of the fetal abdomen in an axial plane of view? A. 1/4 B. 1/3 C. 1/2 D. 2/3

A. Cranial aspects of the fetus Generally, the left side of the heart perfuses the fetal cranium.

The left side of the heart is responsible for perfusing A. Cranial aspects of the fetus B. Systemic aspects of the fetus C. Placenta D. None of the above

C. In the chorionic cavity between the amnion and the chorion The yolk sac is located adjacent to the embryonic plate in early pregnancy and is located within the chorion.

The location of the yolk sac is A. Inside the umbilical cord B. Inside the amniotic cavity C. In the chorionic cavity between the amnion and the chorion D. Outside the chorionic cavity between the chorion and the endometrial wall.

D. A and B The list of causes for nonimmune hydrops fetalis (NIHF) is more than 120 conditions, some of which are rare. Major causes are: cardiac arrhythmias and tumors, abnormal chromosomes, cardiac failure, anemia, AV shunts, mediastinal compression, metabolic disease, fetal infection, fetal tumors, congenital fetal defects, and placental defects.

The main causes for nonimmune hydrops include A. Fetal abnormalities B. Parvovirus C. Anti-Kell antibodies D. A and B E. A and C F. All of the above

B. False 75% of cephaloceles are occipital.

The majority of cephaloceles are parietal. A. True B. False

A. Left sided Of the diaphragmatic hernias, 75-90% are left sided, 10% are right and 5% are bilateral.

The majority of congenital diaphragmatic hernias are A. Left sided B. Right sided C. Bilateral D. MIdline

B. In the lower left lung base The most common appearance detected prenatally is a well-circumscribed echogenic mass in the left lower lung base.

The majority of pulmonary sequestrations are found a. Inferior to the diaphragm B. In the lower left lung base C. In the lower right lung base D. In the upper left pulmonary lobe

C. 20 mm By approximately day 14 of the menstrual cycle, one or more dominant follicles will reach a mean diameter of 20 mm with a hypoechoic rim.

The mean diameter of a dominant follicular cyst at the time of ovulation is A. 10 mm B. 15 mm C. 20 mm D. 25 mm

A. First trimester ultrasound The CRL is the most accurate because fetal growth is very uniform and is rarely affected by pathological disorders. The choices C and D are based on maternal LMP, which assumes ovulation on day 14.

The most accurate method for establishing EDC is A. First trimester ultrasound B. Second trimester ultrasound C. LMP D. Nagele's rule

D. The "lemon and banana" signs of the fetal cranium Cranial signs are present 99% of the time with spinal defects. The second most accurate technique would be visualizing the spine in the transverse plane of view, looking for vertebral splaying and a break in the skin line.

The most accurate way to diagnosis a spinal defect is A. In the longitudinal plane of the spine B. In the coronal plane of the spine C. In the sagittal plane of the spine D. The "lemon and banana" signs of the fetal cranium

C. Cardiac The risk of significant abnormalities seems to increase with the severity of the cleft. A wide variety of abnormalities are associated with clefts, but cardiac abnormalities are the most prevalent.

The most common associated anatomic abnormalities of cleft lip are A. Genitourinary B. Gastrointestinal C. Cardiac D. There are no associated abnormalities

C. AV canal defect 50% of trisomy 21 cases have a cardiac defect, the majority being AV canal defects.

The most common cardiac defect associated with trisomy 21 is A. Atrial/ventricular septal defect B. Hypoplastic left heart syndrome C. AV canal defect D. Transposition of the great vessels

B. Atrial/ventricular septal defect Atrial and ventricular septal defects are the most common defects accounting for 26% of the cardiac abnormalities.

The most common cardiac defect is A. Hypoplastic left heart syndrome B. Atrial/ventricular septal defect C. Tetralogy of Fallot D. None of the above

A. Ovarian tumors Ovarian tumors account for 50-81% of torsion.

The most common cause for ovarian torsion is A. Ovarian tumors B. Hyper stimulation of the ovary C. Adhesions D. Hypervascularity to the ovary

A. Anterior cephalocele The most common cause of hypertelorism is a defect that prevents the migration of the eyes to their normal position. An anterior cephalocele is the most common blockage of that migration.

The most common cause of hypertelorism is A. Anterior cephalocele B. Holoprosencephaly C. Hydranencephaly D. Craniosynostosis

D. Holoprosencephaly The most common cause of hypotelorism is holoprosencephaly. Hypotelorism is found in many different syndromes and chromosomal abnormalities and is strongly associated with abnormalities of the brain.

The most common cause of hypotelorism is A. Dandy-Walker malformation B. Arnold-Chiari type II C. Goldenhar syndrome D. Holoprosencephaly

C. Beckwith-Wiedemann syndrome Macroglossia is present in 97.5% of Beckwith-Wiedemann syndrome.

The most common cause of macroglossia is A. Micrognathia B. Trisomy 18 C. Beckwith-Wiedemann syndrome D. Obstruction of the fetal airway

C. Trisomy 13 Thirty to fifty percent of fetuses with holoprosenecphaly have chromosomal abnormalities, the most common being trisomy 13.

The most common chromosomal abnormality associated with holoprosencephaly is A. Trisomy 21 B. Trisomy 18 C. Trisomy 13 D. Turner syndrome

E. All of the above Hemorrhage, thromboembolism, and infection are the most common complications during the postpartum period. Placenta previa is an antepartum complication.

The most common complication during the postpartum period is A. Hemorrhage B. Thromboembolism C. Infection D. A and B E. All of the above

D. Cytomegalovirus With a maternal CMV infection, the risk of transmission to the fetus is 40-50%, regardless of gestational age.

The most common intrauterine viral infection is A. Toxoplasmosis B. HIV C. Parvovirus D. Cytomegalovirus

D. Superior to the uterine fundus

The most common location for a dermoid cyst is A. posterior cul-de-sac B. Right adnexa C. Left adnexa D. Superior to the uterine fundus

B. Thanatophoric dysplasia The occurrence rate for thanatophoric dysplasia is 1/6000-1/17,000 births. Sonographic findings are polyhydramnios, severe rhizomelia, and micromelia with bowing. The thorax is bell-shaped, and the cranium is cloverleaf-shaped with hydrocephaly and frontal bossing.

The most common short limb syndrome is: A. Achondrogenesis B. Thanatophoric dysplasia C. Osteogenesis imperfecta D. Jeune thoracic dystrophy

C. Dichorionic/diamniotic Seventy -five percent of twins are dizygotic.

The most common twin zygocity is: A. Conjoined twins B. Monochorionic /diamniotic C. Dichorionic/diamniotic D. Monochorionic/ monoamniotic

B. Unilateral cleft lip and palate Of the isolated cleft lip and palate cases, 40% are unilateral cleft lip and palate, 29% are unilateral cleft lip, 27% are bilateral cleft lip and palate, and 5% are bilateral cleft lip.

The most common type of isolated cleft lip/palate is A. Unilateral cleft lip B. Unilateral cleft lip and palate C. Bilateral cleft lip D. Bilateral cleft lip and palate

C. Aqueductal stenosis The order of occurrence of obstruction from most common to least common is: aqueductal stenosis, communicating hydrocephaly, Dandy-Walker malformation, congenital hydrocephaly

The most common type of obstruction causing ventriculomegaly is A. Dandy-Walker malformation B. Communicating hydrocephalus C. Aqueductal stenosis D. Congenital hydrocephaly

A. Leiomyoma Leiomyomas are present in 25% of the female population with a higher percentage in African American women and becoming more prevalent in the fourth generation of life.

The most common uterine tumor is A. Leiomyoma B. Adenomyosis C. Leiomyosarcoma D. Endometrial hyperplasia

D. Iliopsoas muscle The iliopsoas muscle runs laterally to the uterus, near the iliac vessels. In transverse, it is ovoid and hypoechoic and may be mistaken for an ovary.

The most commonly visualized pelvic muscle often mistaken for an ovary is A. Periformis muscle B. Levator ani muscle C. Coccygeus muscle D. Iliopsoas muscle

A. Pouch of Douglas The most dependent portion is the pouch of Douglas, or the posterior cul-de-sac. It is located posterior to the cervix and anterior to the rectum.

The most dependent portion of the peritoneum is called A. Pouch of Douglas B. Vesicouterine pouch C. Retropubic space D. None of the above

C. Congenital cystic adenomatoid malformation CCAM accounts for 75-80% of congenital lung malformations with over 95% of those being unilateral.

The most frequently identified chest mass is A. Right-sided diaphragmatic hernia B. Left-sided diaphragmatic hernia C. Congenital cystic adenomatoid malformation D. Pulmonary sequestration

C. "Lemon"-sized skull The image is an example of a "lemon"-shaped skull. It is most commonly associated with spinal defects, but can be present with encephaloceles and in normal fetuses 1-2% of the time.

The most likely diagnosis for this image includes: A. Strawberry skull B. Down syndrome C. "Lemon"- shaped skull D. Spina bifida E. A and D F. C and D

C. All of the above Nonspecific signs of fetal death are double contour oft he fetal head caused by scalp edema, absence of the falx cerebri because of liquefaction oft he brain, echoes in the amniotic fluid because of fragmentation of the fetal skin, and a decrease in biparietal diameter measurements because of collapse of the cranial sutures after death.

The nonspecific signs of fetal death are which of he following: 1. Echoes in the amniotic fluid 2. The absence of the falx cerebri 3. A decrease in the biparietal diameter (BPD) measurements 4. A double contour of the fetal head (sonographic halo sign) A. 3 and 4 B. 4 only C. All of the above D. None of the above

A. 3 x 2 x 2 cm

The normal adult ovaries measure A. 3 x 2 x 2 cm B. 2 x 2 x 4 cm C. 3 x 3 x 2 cm D. 3 x 2 x 1 cm

B. Double in 48 hours The hCG doubles in approximately 48 hours until 6 weeks.

The normal rise of hCG in a viable pregnancy should A. Double in 24 hours B. Double in 48 hours C. Double in 72 hours D. Double in 1 week

C. 10 x 6 x 5 cm A nulliparous uterus is 8 x 5 x 4 cm. A post menopausal uterus is 7 x 4 x 3 cm

The normal size of a multiparous uterus is A. 5 x 4 x 3 cm B. 7 x 5 x 4 cm C. 10 x 6 x 5 cm D. 6 x 4 x 3 cm

C. The level of the TCD in the axial plane The image should also show the cavum septum pellucidum. The measurement is taken from the outer edge of the cranium to the outer edge of the skin.

The nuchal skin fold in the second trimester should be measured at what level? A. The level of the BPD B. The same image as the OOD C. The level of the TCD in the axial plane D. The level of the ventricles

B. False Amniotic fluid also controls temperature, protects the fetus, allows growth, and prevents infection.

The only purpose of amniotic fluid is to allow for fetal movement. A. True B. False

B. 18-24 weeks Cardiac imaging can be done earlier than 18 weeks, but is dependent on maternal habitus. After 24 weeks, the fetal bones become denser and more calcified and begin to limit the sonographic windows that allow visualization of the cardiac structures.

The optimal time for imaging cardiac structures is A. 14-18 weeks B. 18-24 weeks C. 24-28 weeks D. 28-32 weeks

C. 11 weeks The bowel begins to herniate at 8 weeks and becomes most visible at 9-10 weeks on ultrasound, because of the rotation of the bowel. The bowel then returns to the abdomen by the end of the 11th week.

The physiological herniation of bowel into the umbilical cord is complete by what gestational age? A. 8 weeks B. 9 weeks C. 11 weeks D. 13 weeks

B. Hematopoiesis Hematopoiesis, or red blood cell production, is done by the fetus. The placenta is responsible for the exchange of nutrients, oxygen, and waste. The placenta also acts as a barrier although some medication can cross through the placenta.

The placenta is responsible for all of the following except A. Exchange of nutrients B. Hematopoiesis C. Oxygen exchange D. Barrier to some medications

D. Pulmonary hypoplasia The primary cause of neonatal death in PUV syndrome is pulmonary hypoplasia, although the other entities are also serious complications.

The primary cause of death in posterior urethral valve outlet syndrome is A. Renal failure B. Sepsis C. Cardiac overload D. Pulmonary hypoplasia

A. Placenta previa The incidence of placenta previa at term is 0.5-1%, with 90% of previas bleeding before 38 weeks.

The primary cause of third trimester bleeding is: A. Placenta previa B. Preterm labor C. Cervical bleeding D. Abruption

12-24 weeks = C. head larger than abdomen 32-36 weeks = B. head and body are equal 36-40 weeks = A. Abdomen larger than head

The ratio of head circumference to body circumference normally changes as pregnancy progresses. Match the weeks of gestation in Column A with the head and body ratio in Column B. Column A 12-24 weeks 32-36 weeks 36-40 weeks Column B A. Abdomen larger than head B. Head and body are equal C. Head larger than abdomen

B. Systemic aspects of the fetus Generally, the right side of the heart perfuses the systemic circulation of the fetus.

The right side of the heart is responsible for perfusing A. Cranial aspects of the fetus B. Systemic aspects of the fetus C. Placenta D. None of the above

B. False The data do not support an increased risk attributable to size or number of choroid plexus cysts.

The risk of aneuploidy is increased in cases where the choroid plexus cysts are multiple, bilateral, and/or large. A. True B. False

B. 45° with apex pointed to the left

The rotation of the heart in the fetal chest should be A. 45° with apex pointed to the right B. 45° with apex pointed to the left C. 60° with apex pointed to the right D. The heart should not be rotated in fetal chest

B. False The prognosis of hydrocephaly remains unclear from the prenatal imaging.

The severity of impairment in a child with hydrocephaly may be predicted by the severity of their fetal hydrocephaly. A. True B. False

D. Pulmonary hypoplasia Although chromosomal abnormalities and associated anomalies are prevalent in congenital diaphragmatic hernia, the significant factor in mortality is pulmonary hypoplasia.

The significant perinatal mortality of fetuses with congenital diaphragmatic hernia is caused by A. Chromosomal abnormalities B. Oligohydramnios C. Associated cardiac defects D. Pulmonary hypoplasia

D. Absence of sac within the uterus The decidual reaction may resemble an early intrauterine sac. A yolk sac or embryonic pole will confirm an intrauterine pregnancy.

The sonographic appearance of an ectopic gestation includes all of the following except A. Increased endometrial lining B. Irregular adnexal mass or ovarian mass with ring of color flow Doppler C. Fluid in the cul-de-sac D. Absence of sac within the uterus

D. All of the above Torsed ovaries will appear edematous with a change in blood flow patterns. First is the loss of venous flow, followed by the lack of arterial flow in a complete ovarian torsion.

The sonographic characteristic of ovarian torsion are A. Enlarged ovaries B. Edematous ovarian walls C. Decreased intraovarian blood flow D. All of the above

D. A and C Equinovarus, or clubfoot, may be isolated or associated with other defects, most commonly neural tube defects.

The sonographic finding in this image is: A. Equinovarus B. Normal foot posture C. An association with spina bifida D. A and C E. All of the above

A. Fetal abdominal ascites Abdominal ascites outlines the abdominal viscera. Ascites associated with meconium peritonitis may have particles of debris within it, and echogenic foci are often present in the fetal liver. Nonimmune hydrops must have two fluid collections or one fluid collection with anasarca.

The sonographic finding in this image is: A. Fetal abdominal ascites B. Meconium peritonitis C. Posterior urethral valve obstruction D. Nonimmune hydrops

C. Anencephaly The fetal calvarium is absent superior to the eyes. This finding is consistent with anencephaly.

The sonographic finding in this image is: A. Normal fetal face B. Cleft lip C. Anencephaly D. Hypertelorism

E. All of the above Open tube defect, anencephaly, and cephalocele are part of the neural tube defect spectrum.

The term "neural tube defect" refers to A. Spinal defect B. Open tube defect C. Anencephaly D. Cephalocele E. All of the above

C. Three regions called decidua basalis, decidua perietalis, and decidua capsularis. This marked hypertrophic change in the endometrium occurs no matter where the pregnancy is located. The uterine mucosa responds by a decidual reaction caused by hormonal stimuli. However, when an ectopic pregnancy occurs, the uterine decidua responds by a cast off - decidual cast. This should not be confused with the normal decidua in an early pregnancy.

The term decidua denotes the transformed endometrium of pregnancy. The different regions of the decidua are divided into A. Two regions called decidua basalis and chorionic villi B. One region called decidual reaction C. Three regions called decidua basalis, decidua perietalis, and decidua capsularis D. Three regions called endoderm, mesoderm, and ectoderm.

A. The extra-embryonic peripheral cells of the blastocyst The trophoblast forms these cells, which form the wall of the blastocyst.

The term trophoblast denotes A. The extra-embryonic peripheral cells of the blastocyst B. A rigid state of the flagellate microorganism C. The gestational sac D. The characteristics of a disease

B. Ectopia cordis Ectopia cordis results from underdevelopment or agenesis of the fetal sternum. It is rarely isolated and is most commonly part of the pentalogy of Cantrell.

The terminology for a midline defect of the chest with the heart herniated to the outside of the chest is A. Rachyschisis B. Ectopia cordis C. Cloacal extrophy D. Limb-body wall complex

D. Placenta vessels crossing the internal os. Vasa previa may also be described as a succenturiate placenta with vessels crossing the internal os.

The terminology vasa previa describes: A. Placenta near the internal os B. Placenta touching the internal os C. Placenta crossing the internal os D. Placenta vessels crossing the internal os

C. PCDS, ACDS, and prevesical The posterior cul-de-sac (pouch of Douglas) is located posterior to the uterus. The anterior cul-de-sac (vesicouterine pouch) is located anterior to the uterus. The prevesical space (retropubic space) is anterior to the bladder. They form the peritoneal spaces of the pelvic cavity.

The three peritoneal spaces in the pelvic cavity are: (PCDS- posterior Cul-de-sac, ACDS- anterior cul-de-sac) A. PCDS, ACDS, and vesicouterine B. PCDS, pouch of Douglas, ACDS C. PCDS, ACDS, and prevesical D. PCDS, ACDS, and interstitial

C. The region of the bladder containing the orifices of the ureters and urethra. The trigone is located at the inferior portion of the posterior bladder wall.

The trigone refers to A. The point at which the ureter enters the kidney B. The region where the urethra exits the body C. The region of the bladder containing the orifices of the ureters and urethra. D. The region of the bladder containing the orifice of the urethra.

B. Decreases throughout the pregnancy As the pregnancy advances, the placenta becomes less resistive. This allows for more blood and oxygen to reach the growing fetus.

The umbilical cord S/D ratio normally A. Increases throughout the pregnancy B. Decreases throughout the pregnancy C. Remains the same throughout pregnancy D. Is controlled by the fetal cerebellum

D. Velamentous cord insertion A velamentous cord insertion is associated with IUGR, particularly in monochorionic twins.

The umbilical cord inserting in the membranes and coursing to the placenta is termed A. Normal cord insertion B. Succenturiate cord insertion C. Eccentric cord insertion D. Velamentous cord insertion

C. Eccentric cord insertion An eccentric cord insertion is considered a normal variant and is of no clinical significance.

The umbilical cord inserting into the edge of the placenta is called? A. Normal cord insertion B. Marginal cord insertion C. Eccentric cord insertion D. Velamentous cord insertion

B. Serosal, myometrial, endometrial The outer layer is the serosal, or peritoneal layer. The large muscular middle layer is the myometrium, and the inner layer is the endometrium.

The uterine layers are A. Cervix, isthmus, corpus, fundus B. Serosal, myometrial, endometrial C. Fundus, isthmus, corpus, cervix D. Peritoneum, serosal, and myometrial

C. Uterosacral ligament The uterosacral ligament is the distal portion of the cardinal ligament. It anchors the cervix and is responsible for uterine orientation.

The uterine ligament responsible for uterine orientation is A. Transverse ligament B. Broad ligament C. Uterosacral ligament D. Round ligament

B. Retroflexed Retroflexion of the uterus is described as the uterine corpus is flexed posteriorly on the cervix, forming a sharp angle at the cervix.

The uterine position in the iages would best be described as: A. Retroverted B. Retroflexed C. Anteverted d. Anteflexed

A. cervix, isthmus, corpus, fundus The cervix is the most inferior portion of the uterus and invaginates into the vagina. Moving superiorly, the next section is the isthmus beginning at the internal os. The body, or corpus, of the uterus is the largest section of the uterus. The most superior portion is the fundus.

The uterus can be divided into regions. List them from inferior to superior. A. cervix, isthmus, corpus, fundus B. serosal, myometrial, endometrial C. fundus, isthmus, corpus, cervix D. peritoneum, serosal, and myometrial

A. Wharten's jelly Wharten's jelly is a mucoid connective tissue that surrounds the umbilical vein and artery.

The vessels in the umbilical cord are protected by A. Wharten's jelly B. Amniotic fluid C. Serosal fluid D. Myometrium

A. Two arteries, one vein

The vessels of the normal umbilical cord consist of A. Two arteries, one vein B. Two veins, one artery C. One artery, one iliac vein, and the iliac artery D. One artery, one vein

D. Enlarged kidneys The image is consistent with enlarged kidneys. Dysplastic kidneys would have multiple cysts. Infantile polycystic kidney disease is associated with large kidneys; however, they are echogenic and oligohydramnios is present.

This demonstrates what sonographic finding? A. Normal kidneys B. Dysplastic kidneys C. Infantile polycystic kidney disease D. Enlarged kidneys

B. Macroglossia The tongue can be seen protruding from the mouth, consistent with macroglossia. The chin, forehead, and profile are normal.

This image demonstrates what fetal abnormality? A. Micrognathia B. Macroglossia C. Frontal bossing D. Normal fetal profile

B. Meningocele A meningocele is a spina bifida with herniation of the meninges only.

This image demonstrates what fetal anomaly? A. Sacral agenesis B. Meningocele C. Myelomeningocele D. Sacrococcygeal teratoma

B. Posterior urethral bladder obstruction The dilation of the proximal urethra gives the fetal bladder the classic "keyhole"appearance associated with posterior urethral valve obstruction.

This image demonstrates what fetal bladder abnormality? A. Ureterocele B. Posterior urethral valve obstruction C. Cloacal exstrophy D. Normal full bladder

D. Ovarian neoplasm The ovary is large, irregular in contour, and surrounded by ascites. This was a metastatic ovarian neoplasm.

This image demonstrates what ovarian abnormality? A. Ectopic pregnancy B. Corpus luteal cyst C. Polycystic ovarian disease D. Ovarian neoplasm

C. Benign cystic teratoma A differential diagnosis for this mass could be an endometrioma.

This image demonstrates what ovarian abnormality? A. Hemorrhagic corpus luteal cyst B. Tubo-ovarian abscess C. Benign cystic teratoma D. Ovarian torsion

D. None of the above The abnormality is spina bifida of the lumbar spine. The differential may include a sacrococcygeal teratoma based on the image alone, however this fetus had a +lemon/banana sign.

This image demonstrates what sonographic abnormality? A. Hemivertebrae B. Rachischisis C. Sacral agenesis D. None of the above

D. Arnold-Chiari malformation Also known as the "banana sign." It is characterized by the inferior displacement of the cerebellum and has a strong association with spinal defects.

This image demonstrates what sonographic finding? A. Dandy-Walker malformation B. Normal cerebellum C. Arachnoid cyst D. Arnold-Chiari malformation

B. Encephalocele The image shows a defect in the posterior cranial vault. Because brain tissue is herniated through the defect, this would be an occipital encephalocele. This fetus also demonstrates microcephaly caused by the large defect.

This image demonstrates: A. Anencephaly B. Encephalocele C. Cystic hygroma D. Craniosynostosis

A. Cystic hygroma The image represents a cystic hygroma. The multiple septations differentiate it from hydrops. The intact cranium differentiates it from an encephalocele. Cranial findings associated with spina bifida are found within the calvarium.

This image demonstrates: A. Cystic hygroma B. Spina bifida C. Encephalocele D. Nonimmune hydrops

B. Cystic hygroma The septation within the cystic mass is characteristic of a cystic hygroma.

This image demonstrates: A. Nonimmune hydrops B. Cystic hygroma C. Increased nuchal sonolucency D. Normal findings

A. Turner syndrome Seventy-five percent of fetuses with cystic hygromas have a chromosomal abnormality, most commonly Turner Syndrome.

This image is a sonographic marker for: A. Turner syndrome B. Down syndrome C. Trisomy 18 D. Fetal infection

C. Holoprosencephaly Alobar holoprosencephaly is characterized by a single ventricle, single choroid and fused thalamus.

This image is an example of: A. Sonographic artifact B. Severe ventriculomegaly C. Holoprosencephaly D. None of the above

B. Two endometrial linings of a duplicated uterus. The degree of duplication cannot be determined from this image. The interstitial portion of the fallopian tube runs through the cornua, or superior lateral portion of the fundus.

This image is of a transverse plane of view through the fundus of the uterus. The two echogenic lines represent: A. The interstitial portion of the fallopian tube B. Two endometrial linings of a duplicated uterus C. Single endometrial lining D. Arcuate vessels

B. IUD The thin, well-defined, very bright reflection without any through transmission is characteristic of an IUD.

This is a longitudinal plane of view through the midline of the uterus. The echogenic foci represents: A. Endometrial polyp B. IUD C. Endometrial hyperplasia D. Endometrial cancer

D. Adenomyosis The uterus is large with a heterogenous appearance and no definite masses. This finding and the patient's symptoms would be consistent with adenomyosis.

This is a longitudinal uterine image. The patient is G4P4 with complains of menorrhagia and uterine tenderness. The most likely diagnosis would be: A. Normal uterus B. Intramuscular fibroid C. Endometrial hyperplasia D. Adenomyosis

B. Omphalocele Atypically, the bowel is herniated instead of the liver. Notice the membrane around the bowel and the umbilical cord inserting into the membrane.

This is a sonographic example of: A. Gastroschisis B. Omphalocele C. Normal first trimester physiological umbilical cord herniation D. Duodenal atresia

A. Multicystic dysplastic kidney disease Multicystic, dysplastic kidneys have multiple cysts of various sizes. The cysts occur randomly and do not follow any pattern, as with dilated renal pyramids. The parenchyma is usually echogenic.

This is a sonographic example of: A. Multicystic dysplastic kidney disease B. Infantile polycystic kidney disease C. UPJ obstruction D. A and B E. A and C

D. Placenta vasa previa A vasa previa is an accessory placenta with vessels crossing the internal os.

This is a sonographic example of: A. Placenta accreta B. Placental abruption C. Marginal placenta previa D. Placenta vasa previa

E. All of the above Tetralogy of Fallot is comprised of a VSD, overriding aorta, and right ventricular hypertrophy. The hypertrophy is not always present, particularly in the early to mid second trimester. This image shows the VSD and overriding aorta.

This is a sonographic image of what cardiac defect? A. Ventricular septal defect B. Overriding aorta C. Tetralogy of Fallot D. A and C E. All of the above

D. Congenital left diaphragmatic hernia The stomach is located posterior to the fetal heart, which is deviated to the right side of the chest. The solid appearance on the left side of the thorax is consistent with the liver.

This is a sonographic image of: A. Normal fetal thorax and heart B. Congenital cystic adenomatoid malformation of the lung, type III C. Pulmonary sequestration D Congenital left diaphragmatic hernia

B. Increased nuchal skin fold This is an example of increased nuchal skin fold (>6mm). Edema would surround the entire cranium. A cystic hygroma would have a midline septation.

This is a transverse plane of view in the fetal nuchal region. The sonographic finding is: A. Cystic hygroma B. Increased nuchal skin fold C. Fetal edema D. None of the above

A. Rhombencephalon The cystic structure represents the rhombencephalon. It is a normal structure seen between 7-9 weeks and contributes to the fourth ventricle, brain stem, and cerebellum.

This is an image of a fetus at 8 weeks gestational age. The cystic structure within the fetus is: A. Rhombencephalon B. Cystic hygroma C. Increased nuchal translucency D. All of the above

E. A and C The image is an increased nuchal translucency. It has an 80% positive predictive value for trisomy 21. If combined with the first trimester biochemistry, it has a 90% positive predictive value for trisomy 21.

This sonographic image is consistent with what findings? A. 60% positive predictive value for trisomy 21 B. Increased nuchal translucency C. 80% positive predictive value for trisomy 21 D. A and B E. A and C

A 3 month period during gestation.

Trimester

E. Females only The phenotype for Turner syndrome is 45X, indicating only one single X, or female, chromosomes

Turner syndrome affects males and females A. Equally B. Males more than females C. Females more than males D. Males only E. Females only

B. Medium-sized cysts Type II cysts are <2 cm, but still visible

Type II congenital cystic adenomatoid malformation is A. Microcystic B. Medium-sized cysts C. Macrocystic D. Mixed-size cysts

A. Microcystic Sonographically, these appear as a solid, homogenous, echogenic lung mass.

Type III congenital cystic adenomatoid malformation is A. Microcystic B. Medium-sized cysts C. Macrocystic D. MIxed-size cysts

D. Exstrophy of the bladder Other findings of caudal regression syndrome may include renal, gastrointestinal, facial, and spinal abnormalities.

Ultrasound findings in caudal regression syndrome include all of the following except A. Agenesis of the sacrum B. Hypoplasia of the lower extremities C. Agenesis of the coccyx D. Exstrophy of the bladder

A. True Infections, regardless of the type, present in the fetus with the same spectrum of sonographic findings. These may be IUGR, oligohydramnios, ventriculomegaly, microcephaly, and nonimmune hydrops fetalis.

Ultrasound is not able to differentiate between different infections. A. True B. False

B. Chicken pox

Varicella-zoster virus is also known as A. 5ths disease B. Chicken pox C. "Slapped cheek" disease D. Herpes simplex

E. All of the above Other structures to evaluate include atrial sizes, foramen ovale, coronary vessels, thickness of the ventricular walls, cardiac orientation and size.

What anatomy should be evaluated in the four-chamber view? A. Junction of the AV valves with the atrial and ventricular septum intact B. Equally sized ventricular chambers C. Contractility of the heart D. A and B E. All of the above

A. Vein of Galen aneurysm Vein of Galen aneurysm is an AV malformation located posterior to the third ventricle in the midline of the brain.

What cranial finding can cause congestive heart failure and hydrops? A. Vein of Galen aneurysm B. Periventricular leukomalacia C. Dandy-Walker malformation D. Iniencephaly

C. Uterine fundus

What is # 139 pointing to? A. Uterine cervix B. Fimbria C. Uterine fundus D. Right lateral vaginal fornix E. Left lateral vaginal fornix F. Interstitial portion of the Fallopian tube G. Vagina H. Isthmus uteri I. External os of the cervix J. Ovary K. Ovarian ligament L. Uterine corpus M. Fallopian tube N. Infundibulum O. Internal os of cervix P. Isthmus tubae Q. Ampulla R. Endocervical canal

J. Ovary

What is # 141 pointing to? A. Uterine cervix B. Fimbria C. Uterine fundus D. Right lateral vaginal fornix E. Left lateral vaginal fornix F. Interstitial portion of the Fallopian tube G. Vagina H. Isthmus uteri I. External os of the cervix J. Ovary K. Ovarian ligament L. Uterine corpus M. Fallopian tube N. Infundibulum O. Internal os of cervix P. Isthmus tubae Q. Ampulla R. Endocervical canal

B. Fimbria

What is # 143 pointing to? A. Uterine cervix B. Fimbria C. Uterine fundus D. Right lateral vaginal fornix E. Left lateral vaginal fornix F. Interstitial portion of the Fallopian tube G. Vagina H. Isthmus uteri I. External os of the cervix J. Ovary K. Ovarian ligament L. Uterine corpus M. Fallopian tube N. Infundibulum O. Internal os of cervix P. Isthmus tubae Q. Ampulla R. Endocervical canal

E. Left lateral vaginal fornix

What is # 145 pointing to? A. Uterine cervix B. Fimbria C. Uterine fundus D. Right lateral vaginal fornix E. Left lateral vaginal fornix F. Interstitial portion of the Fallopian tube G. Vagina H. Isthmus uteri I. External os of the cervix J. Ovary K. Ovarian ligament L. Uterine corpus M. Fallopian tube N. Infundibulum O. Internal os of cervix P. Isthmus tubae Q. Ampulla R. Endocervical canal

Q. Ampulla

What is # 152 pointing to? A. Uterine cervix B. Fimbria C. Uterine fundus D. Right lateral vaginal fornix E. Left lateral vaginal fornix F. Interstitial portion of the Fallopian tube G. Vagina H. Isthmus uteri I. External os of the cervix J. Ovary K. Ovarian ligament L. Uterine corpus M. Fallopian tube N. Infundibulum O. Internal os of cervix P. Isthmus tubae Q. Ampulla R. Endocervical canal

G. Decidua basalis

What is # 260 pointing to A. Isthmus uteri B. Chorion frondosum C. Decidua parietalis (vera) D. Decidua capsularis E. Cervix F. Amniotic cavity G. Decidua basalis H. Unoccupied lumen between the decidual layers I. Mucous plug

B. Chorion frondosum

What is # 261 pointing to? A. Isthmus uteri B. Chorion frondosum C. Decidua parietalis (vera) D. Decidua capsularis E. Cervix F. Amniotic cavity G. Decidua basalis H. Unoccupied lumen between the decidual layers I. Mucous plug

A. Isthmus uteri

What is # 262 pointing to? A. Isthmus uteri B. Chorion frondosum C. Decidua parietalis (vera) D. Decidua capsularis E. Cervix F. Amniotic cavity G. Decidua basalis H. Unoccupied lumen between the decidual layers I. Mucous plug

I. Mucous plug

What is # 263 pointing to? A. Isthmus uteri B. Chorion frondosum C. Decidua parietalis (vera) D. Decidua capsularis E. Cervix F. Amniotic cavity G. Decidua basalis H. Unoccupied lumen between the decidual layers I. Mucous plug

E. Cervix

What is # 264 pointing to? A. Isthmus uteri B. Chorion frondosum C. Decidua parietalis (vera) D. Decidua capsularis E. Cervix F. Amniotic cavity G. Decidua basalis H. Unoccupied lumen between the decidual layers I. Mucous plug

C. Decidua perietalis (vera)

What is # 265 pointing to? A. Isthmus uteri B. Chorion frondosum C. Decidua parietalis (vera) D. Decidua capsularis E. Cervix F. Amniotic cavity G. Decidua basalis H. Unoccupied lumen between the decidual layers I. Mucous plug

F. Amniotic cavity

What is # 266 pointing to? A. Isthmus uteri B. Chorion frondosum C. Decidua parietalis (vera) D. Decidua capsularis E. Cervix F. Amniotic cavity G. Decidua basalis H. Unoccupied lumen between the decidual layers I. Mucous plug

H. Unoccupied lumen between the decidual layers.

What is # 268 pointing to? A. Isthmus uteri B. Chorion frondosum C. Decidua parietalis (vera) D. Decidua capsularis E. Cervix F. Amniotic cavity G. Decidua basalis H. Unoccupied lumen between the decidual layers I. Mucous plug

K. Ovarian ligament

What is #140 pointing to? A. Uterine cervix B. Fimbria C. Uterine fundus D. Right lateral vaginal fornix E. Left lateral vaginal fornix F. Interstitial portion of the Fallopian tube G. Vagina H. Isthmus uteri I. External os of the cervix J. Ovary K. Ovarian ligament L. Uterine corpus M. Fallopian tube N. Infundibulum O. Internal os of cervix P. Isthmus tubae Q. Ampulla R. Endocervical canal

I. External os of the cervix

What is #142 pointing to? A. Uterine cervix B. Fimbria C. Uterine fundus D. Right lateral vaginal fornix E. Left lateral vaginal fornix F. Interstitial portion of the Fallopian tube G. Vagina H. Isthmus uteri I. External os of the cervix J. Ovary K. Ovarian ligament L. Uterine corpus M. Fallopian tube N. Infundibulum O. Internal os of cervix P. Isthmus tubae Q. Ampulla R. Endocervical canal

F. Interstitial portion of the Fallopian tube

What is #144 pointing to? A. Uterine cervix B. Fimbria C. Uterine fundus D. Right lateral vaginal fornix E. Left lateral vaginal fornix F. Interstitial portion of the Fallopian tube G. Vagina H. Isthmus uteri I. External os of the cervix J. Ovary K. Ovarian ligament L. Uterine corpus M. Fallopian tube N. Infundibulum O. Internal os of cervix P. Isthmus tubae Q. Ampulla R. Endocervical canal

D. Right lateral vaginal fornix

What is #146 pointing to? A. Uterine cervix B. Fimbria C. Uterine fundus D. Right lateral vaginal fornix E. Left lateral vaginal fornix F. Interstitial portion of the Fallopian tube G. Vagina H. Isthmus uteri I. External os of the cervix J. Ovary K. Ovarian ligament L. Uterine corpus M. Fallopian tube N. Infundibulum O. Internal os of cervix P. Isthmus tubae Q. Ampulla R. Endocervical canal

G. Vagina

What is #147 pointing to? A. Uterine cervix B. Fimbria C. Uterine fundus D. Right lateral vaginal fornix E. Left lateral vaginal fornix F. Interstitial portion of the Fallopian tube G. Vagina H. Isthmus uteri I. External os of the cervix J. Ovary K. Ovarian ligament L. Uterine corpus M. Fallopian tube N. Infundibulum O. Internal os of cervix P. Isthmus tubae Q. Ampulla R. Endocervical canal

L. Uterine corpus

What is #148 pointing to? A. Uterine cervix B. Fimbria C. Uterine fundus D. Right lateral vaginal fornix E. Left lateral vaginal fornix F. Interstitial portion of the Fallopian tube G. Vagina H. Isthmus uteri I. External os of the cervix J. Ovary K. Ovarian ligament L. Uterine corpus M. Fallopian tube N. Infundibulum O. Internal os of cervix P. Isthmus tubae Q. Ampulla R. Endocervical canal

H. Isthmus uteri

What is #149 pointing to? A. Uterine cervix B. Fimbria C. Uterine fundus D. Right lateral vaginal fornix E. Left lateral vaginal fornix F. Interstitial portion of the Fallopian tube G. Vagina H. Isthmus uteri I. External os of the cervix J. Ovary K. Ovarian ligament L. Uterine corpus M. Fallopian tube N. Infundibulum O. Internal os of cervix P. Isthmus tubae Q. Ampulla R. Endocervical canal

A. Uterine cervix

What is #150 pointing to? A. Uterine cervix B. Fimbria C. Uterine fundus D. Right lateral vaginal fornix E. Left lateral vaginal fornix F. Interstitial portion of the Fallopian tube G. Vagina H. Isthmus uteri I. External os of the cervix J. Ovary K. Ovarian ligament L. Uterine corpus M. Fallopian tube N. Infundibulum O. Internal os of cervix P. Isthmus tubae Q. Ampulla R. Endocervical canals

P. Isthmus tubae

What is #151 pointing to? A. Uterine cervix B. Fimbria C. Uterine fundus D. Right lateral vaginal fornix E. Left lateral vaginal fornix F. Interstitial portion of the Fallopian tube G. Vagina H. Isthmus uteri I. External os of the cervix J. Ovary K. Ovarian ligament L. Uterine corpus M. Fallopian tube N. Infundibulum O. Internal os of cervix P. Isthmus tubae Q. Ampulla R. Endocervical canal

N. Infundibulum

What is #153 pointing to? A. Uterine cervix B. Fimbria C. Uterine fundus D. Right lateral vaginal fornix E. Left lateral vaginal fornix F. Interstitial portion of the Fallopian tube G. Vagina H. Isthmus uteri I. External os of the cervix J. Ovary K. Ovarian ligament L. Uterine corpus M. Fallopian tube N. Infundibulum O. Internal os of cervix P. Isthmus tubae Q. Ampulla R. Endocervical canal

M. Fallopian tube

What is #154 pointing to? A. Uterine cervix B. Fimbria C. Uterine fundus D. Right lateral vaginal fornix E. Left lateral vaginal fornix F. Interstitial portion of the Fallopian tube G. Vagina H. Isthmus uteri I. External os of the cervix J. Ovary K. Ovarian ligament L. Uterine corpus M. Fallopian tube N. Infundibulum O. Internal os of cervix P. Isthmus tubae Q. Ampulla R. Endocervical canal

R. Endocervical canal

What is #155 pointing to? A. Uterine cervix B. Fimbria C. Uterine fundus D. Right lateral vaginal fornix E. Left lateral vaginal fornix F. Interstitial portion of the Fallopian tube G. Vagina H. Isthmus uteri I. External os of the cervix J. Ovary K. Ovarian ligament L. Uterine corpus M. Fallopian tube N. Infundibulum O. Internal os of cervix P. Isthmus tubae Q. Ampulla R. Endocervical canal

O. Internal os of cervix

What is #156 pointing to? A. Uterine cervix B. Fimbria C. Uterine fundus D. Right lateral vaginal fornix E. Left lateral vaginal fornix F. Interstitial portion of the Fallopian tube G. Vagina H. Isthmus uteri I. External os of the cervix J. Ovary K. Ovarian ligament L. Uterine corpus M. Fallopian tube N. Infundibulum O. Internal os of cervix P. Isthmus tubae Q. Ampulla R. Endocervical canal

D. Decidua capsularis

What is #267 pointing to? A. Isthmus uteri B. Chorion frondosum C. Decidua parietalis (vera) D. Decidua capsularis E. Cervix F. Amniotic cavity G. Decidua basalis H. Unoccupied lumen between the decidual layers I. Mucous plug

C. When the buttocks descend first, the knees are flexed, baby sitting cross-legged

What is complete breech? A. When both feet are prolapsed into the lower uterine segment B. When both feet are prolapsed into the vagina C. When the buttocks descend first, the knees are flexed, baby sitting cross-legged. D. When the thighs and legs are extended upward along the anterior fetal trunk.

A. When one or both feet are prolapsed into the lower uterine segment

What is footling breech? A. When one or both feet are prolapsed into the lower uterine segment B. When one foot is prolapsed into the fundus C. When the buttocks descend first, the knees are flexed, fetus sitting cross-legged. D. When the thighs and legs are extended upward along the anterior fetal trunk

D. When the buttocks descend first, the thighs and legs are extended upward along the anterior fetal trunk

What is frank breech? A. When both feet are prolapsed into the lower uterine segment B. When on foot is prolapsed into the vagina C. When the fetal head is to the maternal right and the buttocks are to the maternal left D. When the buttocks descend first, the thighs and legs are extended upward along the anterior fetal trunk

J. Umbilical arteries

What is structure is # 269 pointing to? A. Placenta B. Inferior vena cava C. Portal sinus D. Umbilical vein E. Aorta F. Left hepatic vein G. Right hepatic vein H. Ductus venosus I. Portal vein J. Umbilical arteries

B. Inferior vena cava

What is structure is # 270 pointing to? A. Placenta B. Inferior vena cava C. Portal sinus D. Umbilical vein E. Aorta F. Left hepatic vein G. Right hepatic vein H. Ductus venosus I. Portal vein J. Umbilical arteries

F. Left hepatic vein

What is structure is # 271 pointing to? A. Placenta B. Inferior vena cava C. Portal sinus D. Umbilical vein E. Aorta F. Left hepatic vein G. Right hepatic vein H. Ductus venosus I. Portal vein J. Umbilical arteries

G. Right hepatic vein

What is structure is # 272 pointing to? A. Placenta B. Inferior vena cava C. Portal sinus D. Umbilical vein E. Aorta F. Left hepatic vein G. Right hepatic vein H. Ductus venosus I. Portal vein J. Umbilical arteries

H. Ductus venosus

What is structure is # 273 pointing to? A. Placenta B. Inferior vena cava C. Portal sinus D. Umbilical vein E. Aorta F. Left hepatic vein G. Right hepatic vein H. Ductus venosus I. Portal vein J. Umbilical arteries

D. Umbilical vein

What is structure is # 274 pointing to? A. Placenta B. Inferior vena cava C. Portal sinus D. Umbilical vein E. Aorta F. Left hepatic vein G. Right hepatic vein H. Ductus venosus I. Portal vein J. Umbilical arteries

A. Placenta

What is structure is # 275 pointing to? A. Placenta B. Inferior vena cava C. Portal sinus D. Umbilical vein E. Aorta F. Left hepatic vein G. Right hepatic vein H. Ductus venosus I. Portal vein J. Umbilical arteries

I. Portal vein

What is structure is # 276 pointing to? A. Placenta B. Inferior vena cava C. Portal sinus D. Umbilical vein E. Aorta F. Left hepatic vein G. Right hepatic vein H. Ductus venosus I. Portal vein J. Umbilical arteries

C. Portal sinus

What is structure is # 277 pointing to? A. Placenta B. Inferior vena cava C. Portal sinus D. Umbilical vein E. Aorta F. Left hepatic vein G. Right hepatic vein H. Ductus venosus I. Portal vein J. Umbilical arteries

E. Aorta

What is structure is # 278 pointing to? A. Placenta B. Inferior vena cava C. Portal sinus D. Umbilical vein E. Aorta F. Left hepatic vein G. Right hepatic vein H. Ductus venosus I. Portal vein J. Umbilical arteries

C . Isolated cleft lip/ palate It has a variable prevalence, with Native American being the highest at 1.6/1000 births.

What is the most common congenital facial anomaly? A. Proboscis B. Hypotelorism C. Isolated cleft lip/ palate D. Low set ears

C. Beckwith-Wiedemann syndrome Beckwith-Wiedemann syndrome is a group of disorders including omphalocele, macroglossia, organomegaly, and hemihypertrophy.

What is the most likely diagnosis if the sonographic findings omphalocele, macroglossia, and organomegaly are found in the same fetus? A. Trisomy 18 B. Trisomy 13 C. Beckwith-Wiedemann syndrome D. Finnish nephrosis

A. Truncus arteriosis Truncus arteriosis consists of one outflow tract overriding a VSD. The right ventricular outflow tract is absent. Differential diagnosis includes tetralogy of Fallot with pulmonary atresia. Identifying the pulmonary arteries branching from the main trunk would differentiate the defect.

What is the name of the cardiac abnormality with one outflow tract giving rise to both the pulmonary and aortic branches and associated with a ventricular septal defect? A. Truncus arteriosis B. Double outlet right ventricle C. Tetralogy of Fallot D. Transposition of the great vessels

B. Uterine The urinary system develops closely with the uterine development. Twenty to thirty percent of patients with uterine anomalies also have renal ectopia or agenesis.

What other organ system has a strong correlation with renal abnormalities? A. Cardiac B. Uterine C. Ovarian D. Muscular

D. 85 The detection rate is 85%, but may vary among patients depending on the maternal habitus, fetal position, AFV, ultrasound machine, and expertise of the sonographer and physician.

What percentage of cardiac defects can be detected from the four-chamber view and outflow tracts? A. 50 B. 65 C. 80 D. 85

B. 65 The detection rate is 65%, but may vary among patients depending on the maternal habitus, fetal position, AFV, ultrasound machine, and expertise of the sonographer and physician.

What percentage of cardiac defects can be detected from the four-chamber view? A. 50 B. 65 C. 80 D. 85

C. 75%

What percentage of cephaloceles are occipital? A. 50 B. 60 C. 75 D. 99

D. 90% There are five types of TE fistulas. The comprise 90% of the cases of esophageal atresia

What percentage of esophageal atresias have a tracheoesophageal fistula? A. 25% B. 50% C. 75% D. 90%

C. 4% 25% of fetuses are breech at 28 weeks, 7% at 32 weeks, and 3-4% at term.

What percentage of fetuses are breech at term? A. 25% B. 7% C. 4% D. 1%

D. 80%

What percentage of patients diagnosed with hydatidiform mole will usually follow a benign course? A. 20% B. 10% C. 50% D. 80%

C. Uterine wall <5mm in the lower uterine segment Although adhesions cannot be seen on ultrasound, occasionally changes related to previous cesarean sections may be appreciated. These changes are found in the lower uterine segment and may include thinning, ballooning, and wedge defect. A thin uterine wall is felt to be at increased risk for uterine rupture.

What sonographic findings may help determine mothers that are not VBAC candidates? A. Decreased cervical length B. Large amount of maternal abdominal scarring from previous cesarean section C. Uterine wall <5mm in the lower uterine segment D. Sonogram cannot aid in determining VBAC candidates.

F. B and D Heterozygous achondroplasia accounts for 80% of achondroplasias. It is the most common form of genetic skeletal dysplasia. It is often not identified before 26-27 weeks.

What sonographic findings would be identified in a fetus with heterozygous achondroplasia? A. Hydrocephaly B. Frontal bossing C. "Bell shaped" thorax D. "Trident hand" E. A and C F. B and D G. All of the above

C. 33 weeks The maximum amniotic fluid volume peaks at approximately 33 weeks, and then begins to decline.

When does the amniotic fluid volume peak in pregnancy? A. 20 weeks B. 24 weeks C. 33 weeks D. 38 weeks

C. The fetus closest to the internal os.

When labeling twins, fetus A should be A. The first fetus identified B. The fetus closest to the fundus C. The fetus closest to the internal os D. Any of the above methods is appropriate

B. Include epiphyseal plate Only the shaft of the femur should be measured in femur length, excluding the femoral neck and other epiphyseal calcification centers.

When measuring the femur, the calipers should be placed at A. The outermost edge of the bone B. Include epiphyseal plate C. The diaphysis of the shaft of the femur D. The edges of the entire bone, including the head and neck of the femur

B. False The lateral ventricle should be measured at the widest portion of the atrium from the medial to the lateral wall. It should not exceed 10 mm.

When measuring the lateral ventricle, the anterior portion of the ventricle should always be measured. A. True B. False

D. Pulmonary hypoplasia Pulmonary hypoplasia can be assumed by the small thoracic circumference and anhydramnios, but underdeveloped fetal lungs are not visible by sonography.

Which is not a direct sonographic finding of posterior urethral valve obstruction? A. "Keyhole" bladder B. Anhydramnios C. Hydronephrosis D. Pulmonary hypoplasia

1 The third number after parity represents the number of abortions (spontaneous and elective).

Which numbers in the term "parity G7P3214" correspond to the number of abortions.

3 The first number after parity represents the number of full-term pregnancies.

Which numbers in the term "parity G7P3214" correspond to the number of full-term pregnancies.

4 The last number stands for the number of living children.

Which numbers in the term "parity G7P3214" correspond to the number of living children?

7 G refers to the gravida or number of pregnancies.

Which numbers in the term "parity G7P3214" correspond to the number of pregnancies total.

2 The number of preterm births is the second number after parity.

Which numbers in the term "parity G7P3214" correspond to the number of preterm infants?

C. Has a very good prognosis with 80-90% survival rate Gastroschisis is rarely associated with chromosomal or non-gastrointestinal disorders and has an excellent survival rate.

Which of the following about gastroschisis is true? A. More common with advanced maternal age B. Often associated with other abnormalities C. Has a very good prognosis with 80-90% survival rate D. Has an autosomal recessive inheritance

A. May spontaneously regress Congenital cystic adenomatoid malformation (CCAM) is divided into three subsets; macro, medium, and microcystic. Survival rate combines all types and sizes and is 75-80%. Studies have shown that CCAM regresses 55-69% of the time.

Which of the following describes congenital cystic adenomatoid malformation? A. May spontaneously regress B. Poor survival rate C. Divided into two subsets, solid and cystic D. None of the above

A. Endometriosis The differential diagnoses that may mimic hydatidiform moles are missed abortions, leiomyomas, and hematomas. Endometriosis is endometrial tissue outside of the endometrial lining.

Which of the following diagnoses does not mimic the sonographic characteristic of hydatidiform mole? A. Endometriosis B. Hematoma C. Uterine leiomyoma D. Anembryonic demise

A. Coarctation of the aorta Coarctation of the aorta is a narrowing of the aorta, usually near the ductus arteriosis. It may be very difficult to image depending on the degree of stenosis. Often diagnosis relies on the ventricular discrepancy indicating that a stenosis is present.

Which of the following heart abnormalities are difficult or unable to be identified on fetal echocardiography? A. Coarctation of the aorta B. Ebstein's anomaly C. Double outlet right ventricle D. None of the above

B. Encountered more in women over 40 years Dermoid cysts are more common in younger women.

Which of the following is not true of dermoid tumors? A. May cast an acoustic shadow B. Encountered more in women over 40 years C. Also called benign cystic teratoma D. Most common benign germ cell tumor in the female E. Unilateral in about 80% of cases F. The tumor has elements such as hair, tooth, bone, endoderm, ectoderm, mesoderm, and thyroid glandular tissue. G. May be echogenic because of fat content and sonographically produce "tip of the iceberg" effect.

E. it is a solid benign tumor. Dysgerminoma is a malignant tumor.

Which of the following is not true regarding dysgerminoma? A. It is a solid malignant germ cell tumor of the ovary. B. It is a counterpart of seminoma of the testis. C. It is a relatively uncommon tumor accounting for about 2% of all ovarian cancers. D. 90% are unilateral E. It is a solid benign tumor.

E. All of the above The "lemon" sign is found in 1-2% of normal fetuses.

Which of the following is true about the "lemon" sign and neural tube defects? A. The "lemon" sign is not as accurate as the "banana" sign. B. The "lemon" sign may be present in the normal fetus in the third trimester. C. The "lemon" sign can be artificially produced at the level of the ventricles. D. The "lemon" sign is a predictor for spinal bifida. E. All of the above

E. B and C Paternally derived trisomy 13 presents with a large placenta sometimes termed a partial mole. Less commonly, a dizygotic pregnancy may occur. One fetus results from normal fertilization of one egg and a complete molar pregnancy results from the fertilization of the other egg. In those cases, it is possible for the hydatidiform mole to advance to choriocarcinoma.

Which of the following is true regarding a hydatidiform mole and coexistent fetus? A. Consistent with maternally derived trisomy 13 B. Consistent with paternally derived trisomy 13 C. Can progress to choriocarcinoma D. A and C E. B and C

D. The largest risk of fetal demise is because of cord entanglement. Forty percent of conjoined twins are born stillborn. Fifty-six percent of conjoined twins are thoracoomphalopagus, thoracopagus, and omphalopagus. Polyhydramnios is present 50% of the time. Commonly, there is one umbilical cord that may have an abnormal number of vessels and is shared by the conjoined fetuses.

Which of the following statements about conjoined twins is not true? A. 60% are born alive B. 56% of conjoined twins are fused on the ventral wall. C. Polyhydramnios is commonly present. D. The largest risk of fetal demise is because of cord entanglement.

A. The yolk sac should be included in measurements of CRL. The yolk sac lies in the chorionic cavity between the amnion and chorionic sac. It measures 5 mm to 1 cm. The yolk sac shrinks as pregnancy advances. It should not be measured in the CRL.

Which of the following statements is not true concerning the yolk sac? A. The yolk sac should be included in measurements of CRL B. The yolk sac shrinks as pregnancy advances. C. The yolk sac plays a role in blood development and transfer of nutrients D. The yolk sac is attached to the body stalk and is located between the amnion and chorion.

F. B and C Congenital diaphragmatic hernia is left sided 75-90% of the time. Prognosis is poor, particularly if the liver is herniated into the chest. Five to fifteen percent of congenital diaphragmatic hernias are associated with chromosomal abnormalities, commonly trisomy 18.

Which of the following statements regarding congenital diaphragmatic hernia is true? A. More commonly right sided than left sided B. Carries a poor prognosis C. May be associated with chromosomal abnormalities D. All of the above E. A and C F. B and C

D. Ovaries The ovaries are least likely to be seen on a postpartum pelvic sonogram. This may be because of extrapelvic position of the ovaries caused by the large uterus.

Which of the following structures is most likely not seen in a postpartum pelvic sonogram? A. Uterus B. Endometrial echo C. Vagina D. Ovaries

B. Suspensory and broad ligament Both the suspensory and broad ligaments are folds of peritoneum.

Which two ligaments are not true ligaments? A. Uterosacral and broad ligament B. Suspensory and broad ligament C. Uterosacral and round ligament D. Cardinal and suspensory ligament

D. All of the above Symptoms of endometriosis caused by adhesions include dysmenorrhea, low back pain, dyspareunia (painful sexual intercourse), irregular bleeding and infertility.

Women with endometriosis may have A. Dyspareunia B. Metromennorhagia C. Dysmenorrhea D. All of the above


Set pelajaran terkait

Chapter 2: Exercise & Sport Sciences

View Set

Exam 4 Molecular Genetics Quizlet

View Set

Exam 1 - Principles of Marketing

View Set

Med Surg 1, Exam 2: Ch. 22 & 23 - Integumentary System & Problems

View Set

Biopsych exam 3 (chapter 5.1/5.2)

View Set

Marketing Test #3: Chapter 9 definitions

View Set

Intro to Business Chapter 17 vocab

View Set